You are on page 1of 80

INTRODUCTION T O

10 DIFFERENTIAL
EQUATIONS
10.1 Solving Differential Equations
Preliminary Questions
1. Determine the order of the following differential equations:
(a) x 5 y  = 1 (b) (y  )3 + x = 1
(c) y  + x 4 y  = 2
SOLUTION
(a) The highest order derivative that appears in this equation is a first derivative, so this is a first order equation.
(b) The highest order derivative that appears in this equation is a first derivative, so this is a first order equation.
(c) The highest order derivative that appears in this equation is a third derivative, so this is a third order equation.
2. Is y  = sin x a linear differential equation?
SOLUTION Yes.
3. Give an example of a nonlinear differential equation of the form y  = f (y).
SOLUTION One possibility is y  = y 2 .
4. Can a nonlinear differential equation be separable? If so, give an example.
SOLUTION Yes. An example is y  = y 2 .
5. Give an example of a linear, nonseparable differential equation.
SOLUTION One example is y  + y = x.

Exercises
1. Which of the following differential equations are first-order?
(a) y  = x 2 (b) y  = y 2
(c) (y  )3 + yy  = sin x
(d) x 2 y  e x y = sin y
y
(e) y  + 3y  = (f) yy  + x + y = 0
x
SOLUTION
(a) The highest order derivative that appears in this equation is a first derivative, so this is a first order equation.
(b) The highest order derivative that appears in this equation is a second derivative, so this is not a first order equation.
(c) The highest order derivative that appears in this equation is a first derivative, so this is a first order equation.
(d) The highest order derivative that appears in this equation is a first derivative, so this is a first order equation.
(e) The highest order derivative that appears in this equation is a second derivative, so this is not a first order equation.
(f) The highest order derivative that appears in this equation is a first derivative, so this is a first order equation.
2. Which of the equations in Exercise 1 are linear?
SOLUTION

(a) Linear; (1)y  x 2 = 0.


(b) Not linear; y 2 is not a linear function of y.
(c) Not linear; (y  )3 is not a linear function of y  .
(d) Not linear; sin y is not a linear function of y.
(e) Linear; (1)y  + (3)y  1x y = 0.
(f) Not linear. yy  cannot be expressed as a(x)y (n) .

In Exercises 39, verify that the given function is a solution of the differential equation.
S E C T I O N 10.1 Solving Differential Equations 1017

3. y  8x = 0, y = 4x 2
SOLUTION Let y = 4x 2 . Then y  = 8x and

y  8x = 8x 8x = 0.

4. y  + 8y = 0, y = 4e8x
SOLUTION Let y = 4e8x . Then y  = 32e8x , and

y  + 8y = 32e8x + 8(4e8x ) = 0.

5. yy  + 4x = 0, y = 12 4x 2

SOLUTION Let y = 12 4x 2 . Then

4x
y =  ,
12 4x 2
and
 4x
yy  + 4x = 12 4x 2  + 4x = 4x + 4x = 0.
12 4x 2

6. y  + 4x y = 0, y = 3e2x
2

Let y = 3e2x . Then y  = 12xe2x , and


2 2
SOLUTION

y  + 4x y = 12xe2x + 4x(3e2x ) = 0.
2 2

7. (x 2 1)y  + x y = 0, y = 4(x 2 1)1/2


SOLUTION Let y = 4(x 2 1)1/2 . Then y  = 4x(x 2 1)3/2 , and

(x 2 1)y  + x y = (x 2 1)(4x)(x 2 1)3/2 + 4x(x 2 1)1/2


= 4x(x 2 1)1/2 + 4x(x 2 1)1/2 = 0.

8. y  2x y  + 8y = 0, y = 4x 4 12x 2 + 3
SOLUTION Let y = 4x 4 12x 2 + 3. Then y  = 16x 3 24x, y  = 48x 2 24, and

y  2x y  + 8y = (48x 2 24) 2x(16x 3 24x) + 8(4x 4 12x 2 + 3)


= 48x 2 24 32x 4 + 48x 2 + 32x 4 96x 2 + 24 = 0.

9. y  2y  + 5y = 0, y = e x sin 2x
SOLUTION Let y = e x sin 2x. Then

y  = 2e x cos 2x + e x sin 2x,


y  = 4e x sin 2x + 2e x cos 2x + 2e x cos 2x + e x sin 2x = 3e x sin 2x + 4e x cos 2x,

and

y  2y  + 5y = 3e x sin 2x + 4e x cos 2x 4e x cos 2x 2e x sin 2x + 5e x sin 2x


= (3e x 2e x + 5e x ) sin 2x + (4e x 4e x ) cos 2x = 0.

10. Which of the following equations are separable? Write those that are separable in the form y  = f (x)g(y) (but do
not solve).

(a) x y  + y 2 = 0 (b) 1 x 2 y  = e y sin x
(c) y  = x 2 + y 2 (d) y  = 9 y 2
SOLUTION
1018 C H A P T E R 10 I N T R O D U C T I O N TO D I F F E R E N T I A L E Q U ATI O N S

(a) x y  + y 2 = 0 is separable:

x y + y2 = 0
x y  = y 2
1
y = y2
x

(b) 1 x 2 y  = e y sin x is separable:

1 x 2 y  = e y sin x
sin x
y = e y  .
1 x2

(c) y  = x 2 + y 2 is not separable; y  is already isolated, but is not equal to a product f (x)g(y).
(d) y  = 9 y 2 is separable: y  = (1)(9 y 2 ).
11. Consider the differential equation y  = e y cos x.
(a) Write it as ey d y = cos
 x d x. 
(b) Integrate both sides of ey d y = cos x d x.
(c) Show that y = ln |C sin x| is the general solution.
(d) Find the particular solution satisfying y(0) = 0.
SOLUTION

(a) As y  = dd xy , we regroup:

dy
= e y cos x
dx
ey d y = cos x d x

(b) Integrating both sides of the resulting equation yields

ey + C1 = sin x + C2
ey = sin x + C

(Note that a linear sum of arbitrary constants is another arbitrary constant.)


(c) Solving the last equation for y yields

y = ln | sin x + C|
y = ln |C sin x|

(d) Setting y(0) = 0 in the last equation yields

0 = ln(C)

so that C = 1. Therefore, y(x) = ln |1 sin x|.


12. Verify that x 2 y  + e y = 0 is separable by rewriting it in the form y  = f (x)g(y).
(a) Show that ey d y = x 2 d x.
(b) Show that ey = x 1 + C. 
(c) Verify that y = lnC x 1  is the general solution.
SOLUTION Solving x 2 y  + e y = 0 for y  yields

y  = x 2 e y .

(a) Separating variables in the last equation yields

ey d y = x 2 d x.

(b) Integrating both sides of the result of part (a), we find


 
ey d y = x 2 d x

ey + C1 = x 1 + C2
ey = x 1 + C
S E C T I O N 10.1 Solving Differential Equations 1019

(c) Solving the last expression from part (b) for y, we find
 
y = lnx 1 + C 
 
y = lnC x 1 .

In Exercises 1328, solve using separation of variables.

13. y  = x y 2
SOLUTION Rewrite

dy dy
= x y2 as = x d x.
dx y2
Integrating both sides of this equation yields
 
dy
= x dx
y2
1 1
= x 2 + C.
y 2
Thus,
1
y=1 ,
2x +C
2

where C is an arbitrary constant.


1
14. y  =xy
2
SOLUTION Rewrite

dy 1 dy 1
= xy as = x d x.
dx 2 y 2
Integrating both sides of this equation yields
 
dy 1
= x dx
y 2
1 2
ln |y| = x + C.
4
Solving for y, we find
1 2 1 2
|y| = e 4 x +C = eC e 4 x
1 2 1 2
y = eC e 4 x = Ae 4 x ,

where A = eC is an arbitrary constant.


15. y  = 9y
SOLUTION Rewrite

dy dy
= 9y as = 9 d x.
dx y
Integrating both sides of this equation yields
 
dy
= 9 dx
y
ln |y| = 9x + C.

Solving for y, we find

|y| = e9x+C = eC e9x


y = eC e9x = Ae9x ,

where A = eC is an arbitrary constant.


1020 C H A P T E R 10 I N T R O D U C T I O N TO D I F F E R E N T I A L E Q U ATI O N S

16. y  = 2(4 y)
SOLUTION Rewrite

dy dy
= 2(4 y) as = 2 d x.
dx 4 y
Integrating both sides of this equation yields
 
dy
= 2 dx
4y
ln |4 y| = 2x + C.

Solving for y, we find

ln |4 y| = 2x C
|4 y| = e2xC = eC e2x
y = 4 eC e2x = 4 + Ae2x ,

where A = eC is an arbitrary constant.


dy
17. 2 + 6y + 4 = 0
dx
SOLUTION Rewrite

dy
2 + (6y + 4) = 0
dx
as
dy dy
= (3y + 2) and then = d x.
dx 3y + 2
Integrating both sides of this equation yields
 
dy
= dx
3y + 2
ln |3y + 2| = x + C.

Solving for y, we find

|3y + 2| = ex+C = eC ex
3y + 2 = eC ex
1 2 2
y = eC ex = Aex ,
3 3 3

where A = 13 eC is an arbitrary constant.


dy
18. =2 y
dt
SOLUTION Rewrite

dy dy
=2 y as = 2 dt.
dt y

Integrating both sides of this equation yields


 
dy
= 2 dt
y

2 y = 2t + C.

Solving for y, we find



y =t +C

y = (t + C)2 ,

where C is an arbitrary constant.


S E C T I O N 10.1 Solving Differential Equations 1021

dy
19. te y = 0
dt
SOLUTION Rewrite

dy
= te y as ey d y = t dt.
dt
Integrating both sides of this equation yields
 
ey d y = t dt

1 2
ey = t + C.
2
Solving for y, we find
 
 1 
y = ln  t 2 + C 
2
 
 1 
y = ln C t 2  ,
2
where C is an arbitrary constant.

20. 1 x 2 y  = x y
SOLUTION Rewrite
 dy dy x
1 x2 = xy as =  d x.
dx y 1 x2
Integrating both sides of this equation yields
 
dy x
=  dx
y 1 x2

ln |y| = 1 x 2 + C.

Solving for y, we find



|y| = e 1x +C = eC e 1x
2 2


y = eC e 1x = Ae 1x ,
2 2

where A is an arbitrary constant.


21. y  = y 2 (1 x 2 )
SOLUTION Rewrite

dy dy
= y 2 (1 x 2 ) as = (1 x 2 ) d x.
dx y2
Integrating both sides of this equation yields
 
dy
= (1 x 2 ) d x
y2
1 3
y 1 = x x + C.
3
Solving for y, we find
1 3
y 1 = x x +C
3
1
y= 1 ,
3x x + C
3

where C is an arbitrary constant.


22. yy  = x
1022 C H A P T E R 10 I N T R O D U C T I O N TO D I F F E R E N T I A L E Q U ATI O N S

SOLUTION Rewrite
dy
y =x as y d y = x d x.
dx
Integrating both sides of this equation yields
 
y dy = x dx

1 2 1
y = x 2 + C.
2 2
Solving for y, we find

y 2 = x 2 + 2C

y = x 2 + A,

where A = 2C is an arbitrary constant.


dx
23. (t 2 + 1) = x2 + 1
dt
SOLUTION Rewrite

dx 1 1
(t 2 + 1) = (x 2 + 1) as dx = 2 dt.
dt x2 + 1 t +1
Integrating both sides of this equation yields
 
1 1
dx = dt
x +1
2 t2 + 1
tan1 x = tan1 t + C.

Solving for x, we find


 
x = tan tan1 t + C .

We can simplify this expression by applying the sum formula for the tangent function:

tan(tan1 t) + tan C t + tan C t+A


x= = = ,
1 tan(tan1 t) tan C 1 t tan C 1 At
where A = tan C is an arbitrary constant.
24. (1 + x 2 )y  = x 3 y
SOLUTION Rewrite

dy 1 x3
(1 + x 2 ) = x3 y as dy = d x.
dx y 1 + x2
Integrating both sides of this equation yields
 
1 x3
dy = d x.
y 1 + x2
3
To integrate x 2 , note
1+x

x3 (x 3 + x) x x
= =x .
1 + x2 1 + x2 1 + x2
Thus,
1 2 1
ln |y| = x ln |x 2 + 1| + C
2 2
e x /2
2

|y| = eC 
x2 + 1

e x /2 e x /2
2 2

y = eC  = A ,
x2 + 1 x2 + 1
where A = eC is an arbitrary constant.
S E C T I O N 10.1 Solving Differential Equations 1023

25. y  = x sec y
SOLUTION Rewrite

dy
= x sec y as cos y d y = x d x.
dx
Integrating both sides of this equation yields
 
cos y d y = x dx

1 2
sin y = x + C.
2
Solving for y, we find
 
1 2
y = sin1 x +C ,
2
where C is an arbitrary constant.
dy
26. = tan y
dt
SOLUTION Rewrite

dy
= tan y as cot y d y = dt.
dt
Integrating both sides of this equation yields
 
cos y
dy = dt
sin y
ln |sin y| = t + C.

Solving for y, we have

|sin y| = et+C = eC et
sin y = eC et

y = sin1 Aet ,

where A = eC is an arbitrary constant.


dy
27. = y tan t
dt
SOLUTION Rewrite

dy 1
= y tan t as d y = tan t dt.
dt y
Integrating both sides of this equation yields
 
1
dy = tan t dt
y
ln |y| = ln |sec t| + C.

Solving for y, we find

|y| = eln |sec t|+C = eC |sec t|


y = eC sec t = A sec t,

where A = eC is an arbitrary constant.


dx
28. = t tan x
dt
SOLUTION Rewrite

dx
= t tan x as cot x d x = t dt.
dt
1024 C H A P T E R 10 I N T R O D U C T I O N TO D I F F E R E N T I A L E Q U ATI O N S

Integrating both sides of this equation yields


 
cot x d x = t dt

1 2
ln |sin x| = t + C.
2
Solving for y, we find
1 2 1 2
| sin x| = e 2 t +C = eC e 2 t
1 2
sin x = eC e 2 t
 1 2
x = sin1 Ae 2 t ,

where A = eC is an arbitrary constant.

In Exercises 2941, solve the initial value problem.

29. y  + 2y = 0, y(ln 2) = 3
SOLUTION First, we find the general solution of the differential equation. Rewrite

dy 1
+ 2y = 0 as d y = 2 d x,
dx y
and then integrate to obtain

ln |y| = 2x + C.

Thus,

y = Ae2x ,

where A = eC is an arbitrary constant. The initial condition y(ln 2) = 3 allows us to determine the value of A.
1
3 = Ae2(ln 2) ; 3= A ; so 12 = A.
4
Finally,

y = 12e2x .

30. y  2y + 4 = 0, y(1) = 4
SOLUTION First, we find the general solution of the differential equation. Rewrite

dy 1
2y + 4 = 0 as d y = 1 d x,
dx 2y 4
and then integrate to obtain
1
ln |2y 4| = x + C.
2
Thus,

y = Ae2x + 2,

where A = 12 e2C is an arbitrary constant. The initial condition y(1) = 4 allows us to determine the value of A.

4 = Ae2 + 2; 2 = Ae2 ; so 2e2 = A.

Finally,

y = 2e2 e2x + 2 = 2e2x2 + 2.

31. yy  = xey ,
2
y(0) = 1
S E C T I O N 10.1 Solving Differential Equations 1025

SOLUTION First, we find the general solution of the differential equation. Rewrite
dy
= xey
2 2
y as ye y d y = x d x,
dx
and then integrate to obtain
1 y2 1
e = x 2 + C.
2 2
Thus,

y = ln(x 2 + A),

where A = 2C is an arbitrary
 constant. The initial condition y(0) = 1 allows us to determine the value of A. Since
y(0) < 0, we have y = ln(x 2 + A), and

1 = ln( A); 1 = ln( A); so e = A.

Finally,

y = ln(x 2 + e).

dy
32. y 2 = x 3 , y(2) = 0
dx
SOLUTION First, we find the general solution of the differential equation. Rewrite

dy
y2 = x 3 as y 2 d y = x 3 d x,
dx
and then integrate to obtain
1 3 1
y = x 2 + C.
3 2
Thus,
 
3 2 1/3
y= A x ,
2
where A = 3C is an arbitrary constant. The initial condition y(2) = 0 allows us to determine the value of A.
 1/3  
3 3 1/3 3
0 = A 22 ; 0= A ; so A = .
2 8 8
Finally,
 
3 3 2 1/3
y= x .
8 2

33. y  = (x 1)(y 2), y(0) = 3


SOLUTION First, we find the general solution of the differential equation. Rewrite
dy 1
= (x 1)(y 2) as d y = (x 1) d x,
dx y2
and then integrate to obtain
1 2
ln |y 2| = x x + C.
2
Thus,

y = Ae(1/2)x x + 2,
2

where A = eC is an arbitrary constant. The initial condition y(0) = 3 allows us to determine the value of A.

3 = Ae0 + 2 = A + 2 so A = 1.

Finally,

y = e(1/2)x x + 2.
2
1026 C H A P T E R 10 I N T R O D U C T I O N TO D I F F E R E N T I A L E Q U ATI O N S

dy
34. (1 t) y = 0, y(2) = 4
dt
SOLUTION First, we find the general solution of the differential equation. Rewrite

dy 1 1
(1 t) =y as dy = dt,
dt y t 1
and then integrate to obtain

ln |y| = ln |t 1| + C.

Thus,
A
y= ,
t 1
where A = eC is an arbitrary constant. The initial condition y(2) = 4 allows us to determine the value of A.
A
4 = = A.
21
Finally,
4
y= .
t 1
dy
35. = yet , y(0) = 1
dt
SOLUTION First, we find the general solution of the differential equation. Rewrite

dy 1
= yet as d y = et dt,
dt y
and then integrate to obtain

ln |y| = et + C.

Thus,
t
y = Aee ,

where A = eC is an arbitrary constant. The initial condition y(0) = 1 allows us to determine the value of A.

1 = Ae1 so A = e.

Finally,
t t
y = (e)ee = e1e .

dy
36. = tey , y(1) = 0
dt
SOLUTION First, we find the general solution of the differential equation. Rewrite

dy
= tey as e y d y = t dt,
dt
and then integrate to obtain
1 2
ey = t + C.
2
Thus,
 
1 2
y = ln t +C ,
2
where C is an arbitrary constant. The initial condition y(1) = 0 allows us to determine the value of C.
 
1 1 1
0 = ln + C ; 1 = + C; so C = .
2 2 2
Finally,
 
1 2 1
y = ln t + .
2 2
S E C T I O N 10.1 Solving Differential Equations 1027

dy
37. t 2 t = 1 + y + t y, y(1) = 0
dt
SOLUTION First, we find the general solution of the differential equation. Rewrite

dy
t2 = 1 + t + y + t y = (1 + t)(1 + y)
dt
as
1 1+t
d y = 2 dt,
1+y t
and then integrate to obtain

ln |1 + y| = t 1 + ln |t| + C.

Thus,
t
y = A 1/t 1,
e
where A = eC is an arbitrary constant. The initial condition y(1) = 0 allows us to determine the value of A.
 
1
0= A 1 so A = e.
e
Finally,
et
y = 1/t 1.
e

38. 1 x 2 y  = y 2 + 1, y(0) = 0
SOLUTION First, we find the general solution of the differential equation. Rewrite
 dy 1 1
1 x2 = y2 + 1 as dy =  d x,
dx y2 + 1 1 x2
and then integrate to obtain

tan1 y = sin1 x + C.

Thus,

y = tan(sin1 x + C),

where C is an arbitrary constant. The initial condition y(0) = 0 allows us to determine the value of C.
 
0 = tan sin1 0 + C = tan C so 0 = C.

Finally,
 
y = tan sin1 x .


39. 1 x 2 y = y2, y(0) = 1
SOLUTION First, we find the general solution of the differential equation. Rewrite
 dy dy dx
1 x2 = y2 as 2
=  ,
dx y 1 x2
and then integrate to obtain
1
= sin1 x + C.
y
Thus,
1
y= ,
sin1 x + C
1028 C H A P T E R 10 I N T R O D U C T I O N TO D I F F E R E N T I A L E Q U ATI O N S

where C is an arbitrary constant. The initial condition y(0) = 1 allows us to determine the value of C.
1 1
1= = so C = 1.
sin1 0 + C C
Finally,
1 1
y= = .
sin1 x 1 1 sin1 x


40. y  = tan y, y(ln 2) =
2
SOLUTION First, we find the general solution of the differential equation. Rewrite

dy dy
= tan y as = d x,
dx tan y
and then integrate to obtain

ln |sin y| = x + C.

Thus,

y = sin1 ( Ae x ),

where A = eC is an arbitrary constant. The initial condition y(ln 2) = 2 allows us to determine the value of A.

1
= sin1 (2 A); 1 = 2A so A= .
2 2
Finally,
 
1 x
y = sin1 e .
2

41. y  = y 2 sin x, y(0) = 3


SOLUTION First, we find the general solution of the differential equation. Rewrite

dy
= y 2 sin x as y 2 d y = sin x d x,
dx
and then integrate to obtain

y 1 = cos x + C.

Thus,
1
y= ,
A + cos x
where A = C is an arbitrary constant. The initial condition y(0) = 3 allows us to determine the value of A.
1 1 1 2
3= ; A+1= so A= 1= .
A+1 3 3 3
Finally,
1
y= .
cos x (2/3)

42. Find all values of a such that y = x a is a solution of

y  6x 2 y = 0

SOLUTION Let y = x a . Then

y  = ax a1 and y  = a(a 1)x a2 .

Substituting into the differential equation, we find

y  6x 2 y = a(a 1)x a2 6x a2 = x a2 (a 2 a 6).


S E C T I O N 10.1 Solving Differential Equations 1029

Thus, y  6x 2 y = 0 if and only if

a 2 a 6 = (a 3)(a + 2) = 0.

Hence, y = x a is a solution of the differential equation y  6x 2 y = 0 provided a = 2 or a = 3.


43. Find all values of a such that y = eax is a solution of

y  + 2y  8y = 0

SOLUTION Let y = eax . Then

y  = aeax and y  = a 2 eax .

Substituting into the differential equation, we find

y  + 2y  8y = eax (a 2 + 2a 8).

Because eax is never zero, y  + 2y  8y = 0 if only if a 2 + 2a 8 = (a + 4)(a 2) = 0. Hence, y = eax is a


solution of the differential equation y  + 2y  8y = 0 provided a = 4 or a = 2.
44. Show that if y(t) is a solution of t (y 1)y  = 2y, then t 2 y = Ce y for some constant C [we cannot solve for y(t)
explicitly]. Find t such that y(t) = 2, assuming that y(1) = 1.
SOLUTION Rewrite

dy y1 2
t (y 1) = 2y as d y = dt,
dt y t
and then integrate to obtain

y ln |y| = 2 ln |t| + A.

From here, it follows that


 
y A = ln |y| + ln t 2 = ln |y|t 2

eA e y = |y|t 2
t 2 y = Ce y

where C = eA is an arbitrary constant. The initial condition y(1) = 1 will gives us the value of C.

1(1) = Ce so C = e1 .

Hence, t 2 y = e y1 , and y = 2 when

t 2 (2) = e21
e
t2 =
2

t = e/2.

In Exercises 4548, use Eq. (4) and Torricellis Law [Eq. (5)].

45. A cylindrical tank filled with water has height 10 ft and a base of area 30 ft2 . Water leaks through a hole in the
bottom of area 13 ft2 . How long does it take (a) for half of the water to leak out and (b) for the tank to empty?

SOLUTION Because the tank has a constant cross-sectional area of 30 ft2 and the hole has an area of 13 ft2 , the differ-
ential equation for the height of the water in the tank is
1v
dy v
= 3 = .
dt 30 90
By Torricellis Law,

v = 2gy = 8 y,

using g = 32 ft/s2 . Thus,


dy 4
= y.
dt 45
1030 C H A P T E R 10 I N T R O D U C T I O N TO D I F F E R E N T I A L E Q U ATI O N S

Separating variables and then integrating yields


4
y 1/2 d y = dt
45
4
2y 1/2 = t + C
45
Solving for y, we find
 
2 2
y(t) = C t .
45

Since the tank is originally full, we have the initial condition y(0) = 10, whence 10 = C. Therefore,
 
2 2
y(t) = 10 t .
45
When half of the water is out of the tank, y = 5, so we solve:
 
2 2
5= 10 t
45
for t, finding
45
t= ( 10 5) 20.84 sec.
2
When all of the water is out of the tank, y = 0, so
2 45
10 t = 0 and t = 10 71.15 sec.
45 2

46. A conical tank filled with water has height 12 ft [Figure 7(A)]. Assume that the top is a circle of radius 4 ft and that
water leaks through a hole in the bottom of area 2 in.2 . Let y(t) be the water level at time t.

4
40 ft

12

Hole Radius 10 ft Hole

(A) Conical tank (B) Horizontal tank


FIGURE 7

(a) Show that the cross-sectional area of the tank at height y is A(y) = 9 y 2 .
(b) Find the differential equation satisfied by y(t) and solve for y(t). Use the initial condition y(0) = 12.
(c) How long does it take for the tank to empty?
SOLUTION
(a) By similar triangles, the radius r at height y satisfies
r 4 1
= = ,
y 12 3
so r = y/3 and
2
A(y) = r 2 = y .
9
1 ft2 , so the differential equation for the height of the water in the tank
(b) The area of the hole is B = 2 in2 = 72
becomes:

dy 8B y 1
= = y 3/2 .
dt A(y)
S E C T I O N 10.1 Solving Differential Equations 1031

Separating variables and integrating then yields


dt
y 3/2 d y =

2 5/2 t
y =C
5

When t = 0, y = 12, so we find C = 25 (12)5/2 . Therefore,


 
5t 2/5
y(t) = 125/2 .
2
(c) The tank is empty when y = 0. Using the result from part (b), y = 0 when
2
t= (12)5/2 626 seconds.
5
Thus, it takes roughly 10.5 minutes for the tank to empty.
47. The tank in Figure 7(B) is a cylinder of radius 10 ft and length 40 ft. Assume that the tank is half-filled with water
and that water leaks through a hole in the bottom of area B = 3 in.2 . Determine the water level y(t) and the time te when
the tank is empty.
SOLUTION When the water is at height y over the bottom, the top cross section is a rectangle with length 40 ft, and
with width x satisfying the equation:

(x/2)2 + (y 10)2 = 100.



Thus, x = 2 20y y 2 , and

A(y) = 40x = 80 20y y 2 .

1 ft2 and v = 2gy = 8 y, it follows that


With B = 3 in2 = 48

1 8 y
dy 1
= 48 = .
dt 80 20y y 2 480 20 y

Separating variables and integrating then yields:


 1
20 y d y = dt
480
2 t
(20 y)3/2 = +C
3 480

When t = 0, y = 10, so C = 23 103/2 , and

2 t 2
(20 y)3/2 = 103/2
3 480 3
 2/3
t
y(t) = 20 + 103/2 .
320
The tank is empty when y = 0. Thus, te satisfies the equation
 2/3
te
20 + 103/2 = 0.
320
It follows that

te = 320(203/2 103/2 ) 18502.4 seconds.

48. A cylindrical tank filled with water has height h ft and a base of area A ft2 . Water leaks through a hole in
the bottom of area B ft2 .

(a) Show that the time required for the tank to empty is proportional to A h/B.
(b) Show that the emptying time is proportional to V h 1/2 , where V is the volume of the tank.
(c) Two tanks have the same volume and same-sized hole, but different heights and bases. Which tank empties first: the
taller or shorter tank?
1032 C H A P T E R 10 I N T R O D U C T I O N TO D I F F E R E N T I A L E Q U ATI O N S

SOLUTION Torricellis law gives the differential equation for the height of the water in the tank as

dy  B y
= 2g
dt A
Separating variables and integrating then yields:
 B
y 1/2 d y = 2g dt
A
 Bt
2y 1/2 = 2g +C
A
 Bt
y 1/2 = g/2 +C
A
When t = 0, y = h, so C = h 1/2 and
 Bt
y 1/2 = h g/2 .
A
(a) When the tank is empty, y = 0. Thus, the time required for the tank to empty, te , satisfies the equation
 Bte
0= h g/2 .
A
It follows that

A  A h
te = 2h/g = 2/g ;
B B

that is, the time required for the tank to empty is proportional to A h/B.
(b) The volume of the tank is V = Ah; therefore

A h 1 V
= ,
B B h
and
 
 A h 2/g V
te = 2/g = ;
B B h

that is, the time required for the tank to empty is proportional to V h 1/2 .
(c) By part (b), with V and B held constant, the emptying time decreases with height. The taller tank therefore empties
first.
49. Figure 8 shows a circuit consisting of a resistor of R ohms, a capacitor of C farads, and a battery of voltage V . When
the circuit is completed, the amount of charge q(t) (in coulombs) on the plates of the capacitor varies according to the
differential equation (t in seconds)
dq 1
R + q=V
dt C
(a) Solve for q(t).
(b) Show that lim q(t) = C V .
t
(c) Find q(t), assuming that q(0) = 0. Show that the capacitor charges to approximately 63% of its final value C V after
a time period of length = RC ( is called the time constant of the capacitor).

V C

FIGURE 8 An RC circuit.

SOLUTION
S E C T I O N 10.1 Solving Differential Equations 1033

(a) Upon rearranging the terms of the differential equation, we have


dq q CV
= .
dt RC
Separating the variables and integrating both sides, we obtain
dq dt
=
q CV RC
 
dq dt
=
q CV RC
and
t
ln |q C V | = + k,
RC
where k is an arbitrary constant. Solving for q(t) yields
1
q(t) = C V + K e RC t ,

where K = ek .
(b) Using the result from part (a), we calculate
 
lim q(t) = lim C V + K e RC t = C V + K lim e RC t = C V + K 0 = C V .
1 1

t t t

(c) Using the result from part (a), the condition q(0) = 0 determines K = C V . Thus,
 1

q(t) = C V 1 e RC t ,

and

q( ) = q(RC) = C V (1 e1 ) 0.632C V .

50. Assume in the circuit of Figure 8 that R = 100 , C = 0.01 F, and V = 10 V. How many seconds does it take for
the charge on the capacitor plates to reach half of its limiting value?
SOLUTION From Exercise 49, we know that
 
q(t) = C V 1 et/(RC) = 0.1(1 et ),

and the limiting value of q(t) is C V = 0.1. If the charge on the capacitor plates has reached half its limiting value, then
0.1
= 0.1(1 et )
2
1 et = 1/2
et = 1/2
t = ln 2

Therefore, the charge on the capacitor plates reaches half of its limiting value after ln 2 .693 seconds.
dV
51. One hypothesis for the growth rate of the volume V of a cell is that is proportional to the cells surface
dt
area A. Since V has cubic units such as cm3 and A has square units such as cm2 , we may assume roughly that A V 2/3 ,
dV
and hence = kV 2/3 for some constant k. If this hypothesis is correct, which dependence of volume on time would
dt
we expect to see (again, roughly speaking) in the laboratory?
(a) Linear (b) Quadratic (c) Cubic
SOLUTION Rewrite
dV
= kV 2/3 as V 2/3 dv = k dt,
dt
and then integrate both sides to obtain

3V 1/3 = kt + C
V = (kt/3 + C)3 .

Thus, we expect to see V increasing roughly like the cube of time.


1034 C H A P T E R 10 I N T R O D U C T I O N TO D I F F E R E N T I A L E Q U ATI O N S

52. We might also guess that the rate at which a snowball melts is proportional to its surface area. What is the differential
equation satisfied by the volume V of a spherical snowball at time t? Suppose the snowball has radius 4 cm and that it
loses half of its volume after 10 min. According to this model, when will the snowball disappear?
SOLUTION Because the snowball is spherical, V = 43 r 3 , A = 4 r 2 , and the differential equation

dV
= k A
dt
becomes
dr dr
4 r 2 = k(4 r 2 ) or = k.
dt dt
Thus, r (t) = kt + C, for some arbitrary constant C. Initially, the snowball has radius 4 cm, so C = 4 and r (t) = 4 kt.
Because the initial radius is 4 cm, the initial volume of the snowball is 256
3 cm . After ten minutes, the volume has
3
128

3
been reduced to 3 cm , so the radius has been reduced to 2 4 cm. Therefore,
3


3
2 4 = 4 k(10),

and

2 3 4
k= cm/min.
5
The snowball has disappeared when r = 0. This will occur when 4 kt = 0, or when
4 20
t= = 48.5 minutes.
k 2 34

53. In general, ( f g) is not equal to f  g  , but let f (x) = e2x and find a function g(x) such that ( f g) = f  g  . Do the
same for f (x) = x.
SOLUTION If ( f g) = f  g  , we have

f  (x)g(x) + g  (x) f (x) = f  (x)g  (x)


g  (x)( f (x) f  (x)) = g(x) f  (x)
g  (x) f  (x)
= 
g(x) f (x) f (x)

Now, let f (x) = e2x . Then f  (x) = 2e2x and

g  (x) 2e2x
= 2x = 2.
g(x) 2e e2x
Integrating and solving for g(x), we find
dg
= 2 dx
dg
ln |g| = 2x + C
g(x) = Ae2x ,

where A = eC is an arbitrary constant.


If f (x) = x, then f  (x) = 1, and

g  (x) 1
= .
g(x) 1x
Thus,
dg 1
= dx
g 1x
ln |g| = ln |1 x| + C
A
g(x) = ,
1x

where A = eC is an arbitrary constant.


S E C T I O N 10.1 Solving Differential Equations 1035

54. A boy standing at point B on a dock holds a rope of length  attached to a boat at point A [Figure 9(A)]. As the
boy walks along the dock, holding the rope taut, the boat moves along a curve called a tractrix (from the Latin tractus
meaning to pull). The segment from a point P on the curve to the x-axis along the tangent line has constant length .
Let y = f (x) be the equation of the tractrix.
y
(a) Show that y 2 + (y/y  )2 = 2 and conclude y  =  . Why must we choose the negative square root?
 y2
2
(b) Prove that the tractrix is the graph of

 + 2 y 2
x =  ln 2 y 2
y

A A

P = (x, f(x))


f (x)

x x
B x
Dock f (x)

f ' (x)

FIGURE 9

SOLUTION
(a) From the diagram on the right in Figure 9, we see that
 
f (x) 2
f (x)2 +  = 2 .
f (x)

If we let y = f (x), this last equation reduces to y 2 + (y/y  )2 = 2 . Solving for y  , we find
y
y =  ,
2 y 2
where we must choose the negative sign because y is a decreasing function of x.
(b) Rewrite

dy y 2 y 2
=  as d y = d x,
dx 2 y 2 y

and then integrate both sides to obtain


  2
 y2
x + C = d y.
y
For the remaining integral, we use the trigonometric substitution y =  sin , d y =  cos d . Then
  2   
 y2 cos2 1 sin2
dy =  d =  d =  (csc sin ) d
y sin sin

 2 y 2
=  [ln | csc cot | + cos ] + C =  ln + 2 y 2 + C
y y

Therefore,

2 y 2 y
x =  ln 2 y 2 + C =  ln  2 y 2 + C
y  2 y 2

 + 2 y 2
=  ln 2 y 2 + C
y

Now, when x = 0, y = , so we find C = 0. Finally, the equation for the tractrix is



 + 2 y 2
x =  ln 2 y 2 .
y
1036 C H A P T E R 10 I N T R O D U C T I O N TO D I F F E R E N T I A L E Q U ATI O N S

55. If a bucket of water spins about a vertical axis with constant angular velocity (in radians per second), the water
climbs up the side of the bucket until it reaches an equilibrium position (Figure 10). Two forces act on a particle located
at a distance x from the vertical axis: the gravitational force mg acting downward and the force of the bucket on the

y = f(x)

m 2x

mg
x
x

FIGURE 10

particle (transmitted indirectly through the liquid) in the direction perpendicular to the surface of the water. These two
forces must combine to supply a centripetal force m 2 x, and this occurs if the diagonal of the rectangle in Figure 10 is
normal to the waters surface (that is, perpendicular to the tangent line). Prove that if y = f (x) is the equation of the
curve obtained by taking a vertical cross section through the axis, then 1/y  = g/( 2 x). Show that y = f (x) is a
parabola.
SOLUTION At any point along the surface of the water, the slope of the tangent line is given by the value of y  at that
point; hence, the slope of the line perpendicular to the surface of the water is given by 1/y  . The slope of the resultant
force generated by the gravitational force and the centrifugal force is
mg g
= 2 .
m2 x x
Therefore, the curve obtained by taking a vertical cross-section of the water surface is determined by the equation

1 g 2
 = 2 or y = x.
y x g
Performing one integration yields

2 2
y = f (x) = x + C,
g
where C is a constant of integration. Thus, y = f (x) is a parabola.
3y x
56. Show that the differential equations y  = and y  = define orthogonal families of curves; that is, the
x 3y
graphs of solutions to the first equation intersect the graphs of the solutions to the second equation in right angles (Figure
11). Find these curves explicitly.

FIGURE 11 Two orthogonal families of curves.

SOLUTION Let y1 be a solution to y  = 3y  x


x and let y2 be a solution to y = 3y . Suppose these two curves intersect at
a point (x 0 , y0 ). The line tangent to the curve y1 (x) at (x 0 , y0 ) has a slope of 3y
x0 and the line tangent to the curve y2 (x)
0

x0
has a slope of 3y . The slopes are negative reciprocals of one another; hence the tangent lines are perpendicular.
0
Separation of variables and integration applied to y  = 3y
x gives

dy dx
=3
y x
ln |y| = 3 ln |x| + C
y = Ax 3
S E C T I O N 10.1 Solving Differential Equations 1037

On the other hand, separation of variables and integration applied to y  = 3y


x gives

3y d y = x d x
3y 2 /2 = x 2 /2 + C

y = C x 2 /3

57. Find the family of curves satisfying y  = x/y and sketch several members of the family. Then find the differential
equation for the orthogonal family (see Exercise 56), find its general solution, and add some members of this orthogonal
family to your plot.
SOLUTION Separation of variables and integration applied to y  = x/y gives

y dy = x dx
1 2 1
y = x2 + C
2 2

y = x2 + C

If y(x) is a curve of the family orthogonal to these, it must have tangent lines of slope y/x at every point (x, y). This
gives

y  = y/x

Separation of variables and integration give


dy dx
=
y x
ln |y| = ln |x| + C
A
y=
x
Several solution curves of both differential equations appear below:

"

1 1
1

58. A 50-kg model rocket lifts off by expelling fuel at a rate of k = 4.75 kg/s for 10 s. The fuel leaves the end of the
rocket with an exhaust velocity of b = 100 m/s. Let m(t) be the mass of the rocket at time t. From the law of conservation
of momentum, we find the following differential equation for the rockets velocity v(t) (in meters per second):
dm
m(t)v  (t) = 9.8m(t) + b
dt
(a) Show that m(t) = 50 4.75t kg.
(b) Solve for v(t) and compute the rockets velocity at rocket burnout (after 10 s).
SOLUTION
(a) For 0 t 10, the rocket is expelling fuel at a constant rate of 4.75 kg/s, giving m  (t) = 4.75. Hence, m(t) =
4.75t + C. Initially, the rocket has a mass of 50 kg, so C = 50. Therefore, m(t) = 50 4.75t.
dm
(b) With m(t) = 50 4.75t and = 4.75, the equation for v becomes
dt
dm
dv b dt (100)(4.75)
= 9.8 + = 9.8 +
dt 50 4.75t 50 4.75t
and therefore

4.75 dt
v(t) = 9.8t 100 = 9.8t + 100 ln(50 4.75t) + C
50 4.75t
1038 C H A P T E R 10 I N T R O D U C T I O N TO D I F F E R E N T I A L E Q U ATI O N S

Because v(0) = 0, we find C = 100 ln 50 and

v(t) = 9.8t + 100(ln(50) ln(50 4.75t)).

After 10 seconds the velocity is:

v(10) = 98 + 100(ln(50) ln(2.5)) 201.573 m/s.

59. Let v(t) be the velocity of an object of mass m in free fall near the earths surface. If we assume that air resistance is
dv
proportional to v 2 , then v satisfies the differential equation m = g + kv 2 for some constant k > 0.
dt
(a) Set = (g/k)1/2 and rewrite the differential equation as
dv k
= ( 2 v 2 )
dt m
Then solve using separation of variables with initial condition v(0) = 0.
(b) Show that the terminal velocity lim v(t) is equal to .
t
SOLUTION

(a) Let = (g/k)1/2 . Then


dv g k k g  k  2 
= + v2 = v2 = v2
dt m m m k m
Separating variables and integrating yields
 
dv k k
= dt = t +C
2 v2 m m
We now use partial fraction decomposition for the remaining integral to obtain
    + v
dv 1 1 1  1  
= + dv = ln  
v
2 2 2 +v v 2 v
Therefore,
 
1  + v
ln   = k t + C.
2 v m
The initial condition v(0) = 0 allows us to determine the value of C:
 
1  + 0
ln   = k (0) + C
2 0 m
1
C= ln 1 = 0.
2
Finally, solving for v, we find

1 e2( gk/m)t
v(t) = .
1 + e2( gk/m)t

(b) As t , e2( gk/m)t 0, so
 
10
v(t) = .
1+0

Further Insights and Challenges


60. In Section 6.2, we computed the volume V of a solid as the integral of cross-sectional area. Explain this
formula in terms of differential equations. Let V (y) be the volume of the solid up to height y and let A(y) be the
cross-sectional area at height y as in Figure 12.
(a) Explain the following approximation for small y:

V (y + y) V (y) A(y) y 9


 b
dV
(b) Use Eq. (9) to justify the differential equation = A(y). Then derive the formula V = A(y) d y.
dy a
S E C T I O N 10.1 Solving Differential Equations 1039

b
Area of cross section
y + y is A(y)
y Volume of slice is
V(y + y) V(y) A(y)y

y0 = a

FIGURE 12

SOLUTION
(a) If y is very small, then the slice between y and y + y is very similar to the prism formed by thickening the
cross-sectional area A(y) by a thickness of y. A prism with cross-sectional area A and height y has volume Ay.
This gives

V (y + y) V (y) A(y)y.

(b) Dividing Eq. (9) by y, we obtain


V (y + y) V (y)
A(y).
y
In the limit as y 0, this becomes
dV
= A(y).
dy
Integrating this last equation yields
 b
V = A(y) d y.
a

61. In most cases of interest, the general solution of a differential equation of order n depends on n arbitrary constants.
This exercise shows there are exceptions.
(a) Show that (y  )2 + y 2 = 0 is a first-order equation with only one solution y = 0.
(b) Show that (y  )2 + y 2 + 1 = 0 is a first-order equation with no solutions.
SOLUTION

(a) (y  )2 + y 2 0 and equals zero if and only if y  = 0 and y = 0


(b) (y  )2 + y 2 + 1 1 > 0 for all y  and y, so (y  )2 + y 2 + 1 = 0 has no solution
62. (a) Let P(x) = x 2 + ax + b with a, b constants. Show that y = Ce x (for any constant C) is a solution of
y  + ay  + by = 0 if and only if P( ) = 0.
(b) Show that y = C1 e3x + C2 ex is a solution of y  2y  3y = 0 for any constants C1 , C2 .
SOLUTION

(a) Let y(x) = Ce x . Then y  = Ce x , and y  = 2 Ce x . Thus


 
y  + ay  + by = 2 Ce x + a Ce x + bCe x = Ce x 2 + a + b = Ce x P( ).

Hence, Ce x is a solution of the differential equation y  + ay  + by = 0 if and only if P( ) = 0.


(b) Let y(x) = C1 e3x + C2 ex . Then

y  (x) = 3C1 e3x C2 ex


y  (x) = 9C1 e3x + C2 ex

and

y  2y  3y = 9C1 e3x + C2 ex 6C1 e3x + 2C2 ex 3C1 e3x 3C2 ex


= (9 6 3) C1 e3x + (1 + 2 3) C2 ex = 0.

63. A spherical tank of radius R is half-filled with water. Suppose that water leaks through a hole in the bottom of area
B ft2 . Let y(t) be the water level at time t (seconds).
1040 C H A P T E R 10 I N T R O D U C T I O N TO D I F F E R E N T I A L E Q U ATI O N S

dy 8B y
(a) Show that = .
dt (2Ry y 2 )
(b) Show that for some constant C,
 
2 3/2 1 5/2
Ry y =C t
4B 3 5
(c) Use the initial condition y(0) = R to compute C and show that C = te , the time at which the tank is empty.
(d) Show that te is proportional to R 5/2 and inversely proportional to B.
SOLUTION
(a) At height y above the bottom of the tank, the cross section is a circle of radius

r = R 2 (R y)2 = 2Ry y 2 .

The cross-sectional area function is then A(y) = (2Ry y 2 ). The differential equation for the height of the water in
the tank is then

dy 8B y
= .
dt (2Ry y 2 )
(b) Rewrite the differential equation as
 
2Ry 1/2 y 3/2 d y = dt,
8B
and then integrate both sides to obtain
 
2 3/2 1 5/2
Ry y = C t,
4B 3 5
where C is an arbitrary constant.
(c) The initial condition y(0) = R allows us to determine the value of C:
 
2 5/2 1 5/2 7 5/2
C= R R = R .
4B 3 5 60B
Moreover, note that y = 0 when t = C, C = te , the time at which the tank is empty.
(d) From part (c),
7 5/2
te = R ,
60B
from which it is clear that te is proportional to R 5/2 and inversely proportional to B.

10.2 Graphical and Numerical Methods


Preliminary Questions
1. What is the slope of the segment in the slope field for y = t y + 1 at the point (2, 3)?

SOLUTION The slope of the segment in the slope field for y = t y + 1 at the point (2, 3) is (2)(3) + 1 = 7.

2. What is the equation of the isocline of slope c = 1 for y = y 2 t?



SOLUTION The isocline of slope c = 1 has equation y 2 t = 1, or y = 1 + t.
3. True or false? In the slope field for y = ln y, the slopes at points on a vertical line t = C are all equal.
SOLUTION This statement is true because the right-hand side of the differential equation does not explicitly depend on
the variable t.
4. What about the slope field for y = ln t? Are the slopes at points on a vertical line t = C all equal?
SOLUTION No, because the right-hand side of this differential equation does explicitly depend on the variable t.
5. Let y(t) be the solution to y = F(t, y) with y(1) = 3. How many iterations of Eulers Method are required to
approximate y(3) if the time step is h = 0.1?
SOLUTION The initial condition is specified at t = 1 and we want to obtain an approximation to the value of the
solution at t = 3. With a time step of h = 0.1,
31
= 20
0.1
iterations of Eulers method are required.
S E C T I O N 10.2 Graphical and Numerical Methods 1041

Exercises
1. Figure 8 shows the slope field for y = sin y sin t. Sketch the graphs of the solutions with initial conditions y(0) = 1
and y(0) = 1. Show that y(t) = 0 is a solution and add its graph to the plot.
y
3
2
1
0 t
1
2
3
3 2 1 0 1 2 3
FIGURE 8 Slope field for y = sin y sin t.

SOLUTION The sketches of the solutions appear below.


y
3
2
1
0 t
1
2
3
3 2 1 0 1 2 3

If y(t) = 0, then y  = 0; moreover, sin 0 sin t = 0. Thus, y(t) = 0 is a solution of y = sin y sin t.
2. Figure 9 shows the slope field of y = y 2 t 2 . Sketch the integral curve passing through the point (0, 1), the curve
through (0, 0), and the curve through (0, 2). Is y(t) = 0 a solution?
y
3
2
1
0 t
1
2
3
3 2 1 0 1 2 3
FIGURE 9 Slope field for y = y 2 t 2 .

SOLUTION The sketches of the solutions appear below.


y
3
2
1
0 t
1
2
3
3 2 1 0 1 2 3

Let y(t) = 0. Because y = 0 but y 2 t 2 = t 2


= 0, it follows that y(t) = 0 is not a solution of y = y 2 t 2 .
3. Show that f (t) = 12 (t 12 ) is a solution to y = t 2y. Sketch the four solutions with y(0) = 0.5, 1 on the slope
field in Figure 10. The slope field suggests that every solution approaches f (t) as t . Confirm this by showing that
y = f (t) + Ce2t is the general solution.
y
1
y = 1 (t 1 )
2 2
0.5

0 t

0.5

1
1 0.5 0 0.5 1 1.5 2
FIGURE 10 Slope field for y = t 2y.

1042 C H A P T E R 10 I N T R O D U C T I O N TO D I F F E R E N T I A L E Q U ATI O N S

SOLUTION Let y = f (t) = 12 (t 12 ). Then y = 12 and

y + 2y = + t = t,
1 1
2 2

so f (t) = 12 (t 12 ) is a solution to y = t 2y. The slope field with the four required solutions is shown below.

Now, let y = f (t) + Ce2t = 12 (t 12 ) + Ce2t . Then

y = 2Ce2t ,
1
2
and
 
y + 2y = 2Ce2t + t
1 1
+ 2Ce2t = t.
2 2

Thus, y = f (t) + Ce2t is the general solution to the equation y = t 2y.


4. Consider the differential equation y = t y.
(a) Sketch the slope field of the differential equation y = t y in the range 1 t 3, 1 y 3. As an aid,
observe that the isocline of slope c is the line t y = c, so the segments have slope c at points on the line y = t c.
(b) Show that y = t 1 + Cet is a solution for all C. Since lim et = 0, these solutions approach the particular
t
solution y = t 1 as t . Explain how this behavior is reflected in your slope field.
SOLUTION
(a) Here is a sketch of the slope field:
y
3

0 t

1
1 0 1 2 3

(b) Let y = t 1 + Cet . Then y = 1 C t , and

t y = t (t 1 + Cet ) = 1 Cet .

Thus, y = t 1 + Cet is a solution of y = t y. On the slope field, we can see that the isoclines of 1 all lie along
the line y = t 1. Whenever y > t 1, y = t y < 1, so the solution curve will converge downward towards the
line y = t 1. On the other hand, if y < t 1, y = t y > 1, so the solution curve will converge upward towards
y = t 1. In either case, the solution is approaching t 1.
5. Show that the isoclines of y = 1/y are horizontal lines. Sketch the slope field for 2 t, y 2 and plot the
solutions with initial conditions y(0) = 0 and y(0) = 1.
1 1
SOLUTION The isocline of slope c is defined by y = c. This is equivalent to y = c , which is a horizontal line. The
slope field and the solutions are shown below.
S E C T I O N 10.2 Graphical and Numerical Methods 1043

6. Show that the isoclines of y = t are vertical lines. Sketch the slope field for 2 t, y 2 and plot the integral
curves passing through (0, 1) and (0, 1).

SOLUTION The isocline of slope c for the differential equation y = t has equation t = c, which is the equation of a
vertical line. The slope field and the required solution curves are shown below.
y
2

0 t

2
2 1 0 1 2

7. Sketch the slope field of y = t y for 2 t, y 2. Based on the sketch, determine lim y(t), where y(t) is a
t
solution with y(0) > 0. What is lim y(t) if y(0) < 0?
t
SOLUTION The slope field for y = t y is shown below.
y
2

0 t

2
2 1 0 1 2

With y(0) > 0, the slope field indicates that y is an always increasing, always concave up function; consequently,
limt y = . On the other hand, when y(0) < 0, the slope field indicates that y is an always decreasing, always
concave down function; consequently, limt y = .
8. Match the differential equation with its slope field in Figures 11(A)(F).
(i) y = 1
(ii) y =
y
t
(iii) y = t 2 y
(iv) y = t y 2
(v) y = t 2 + y 2
(vi) y = t

y y
3 3
2 2
1 1
0 t 0 t
1 1
2 2
3 3
3 2 1 0 1 2 3 3 2 1 0 1 2 3
FIGURE 11(A) FIGURE 11(B)
1044 C H A P T E R 10 I N T R O D U C T I O N TO D I F F E R E N T I A L E Q U ATI O N S

y y
3 3
2 2
1 1
0 t 0 t
1 1
2 2
3 3
3 2 1 0 1 2 3 3 2 1 0 1 2 3
FIGURE 11(C) FIGURE 11(D)

y y
3 3
2 2
1 1
0 t 0 t
1 1
2 2
3 3
3 2 1 0 1 2 3 3 2 1 0 1 2 3
FIGURE 11(E) FIGURE 11(F)

SOLUTION
(i) Every segment in the slope field for y = 1 will have slope 1; this matches Figure 11(C).
(ii) The segments in the slope field for y =
y
will have positive slope in the first and third quadrants and negative
t
slopes in the second and fourth quadrant; this matches Figure 11(B).
(iii) The segments in the slope field for y = t 2 y will have positive slope in the upper half of the plane and negative
slopes in the lower half of the plane; this matches Figure 11(F).
(iv) The segments in the slope field for y = t y 2 will have positive slope on the right side of the plane and negative slopes
on the left side of the plane; this matches Figure 11(D).
(v) Every segment in the slope field for y = t 2 + y 2 , except at the origin, will have positive slope; this matches Figure
11(A).
(vi) The isoclines for y = t are vertical lines; this matches Figure 11(E).
9. One of the slope fields in Figures 12(A) and (B) is the slope field for y = t 2 . The other is for y = y 2 . Identify which
is which. In each case, sketch the solutions with initial conditions y(0) = 1, y(0) = 0, and y(0) = 1.

y y
3 3
2 2
1 1
0 t 0 t
1 1
2 2
3 3
3 2 1 0 1 2 3 3 2 1 0 1 2 3
FIGURE 12(A) FIGURE 12(B)

SOLUTION For y  = t 2 , y  only depends on t. The isoclines of any slope c will be the two vertical lines t = c.
This indicates that the slope field will be the one given in Figure 12(A). The solutions are sketched below:
y
2

0 t

2
2 1 0 1 2


For y  = y 2 , y  only depends on y. The isoclines of any slope c will be the two horizontal lines y = c. This indicates
that the slope field will be the one given in Figure 12(B). The solutions are sketched below:
S E C T I O N 10.2 Graphical and Numerical Methods 1045

y
2

0 t

2
2 1 0 1 2

10. (a) Sketch the slope field of y = t/y in the region 2 t 2, 2 y 2.



(b) Check that y = t 2 + C is the general solution.
(c) Sketch the solutions on the slope field with initial conditions y(0) = 1 and y(0) = 1.
SOLUTION
(a) The slope field is shown below:
y
2

0 t

2
2 1 0 1 2

(b) Rewrite
dy t
= as y d y = t dt,
dt y
and then integrate both sides to obtain
1 2 1
y = t 2 + C.
2 2
Solving for y, we find that the general solution is

y = t 2 + C.

(c) The sketches of the two solutions are shown below:


y
2

0 t

2
2 1 0 1 2

11. Sketch the slope field of y = t 2 y in the region 3 t 3, 3 y 3 and sketch the solutions satisfying
y(1) = 0, y(1) = 1, and y(1) = 1.

SOLUTION The slope field for y = t 2 y, together with the required solution curves, is shown below.

y
3
2
1
0 t
1
2
3
3 2 1 0 1 2 3
1046 C H A P T E R 10 I N T R O D U C T I O N TO D I F F E R E N T I A L E Q U ATI O N S

12. Let F(t, y) = t 2 y and let y(t) be the solution of y = F(t, y) satisfying y(2) = 3. Let h = 0.1 be the time step
in Eulers Method and set y0 = y(2) = 3.
(a) Calculate y1 = y0 + h F(2, 3).
(b) Calculate y2 = y1 + h F(2.1, y1 ).
(c) Calculate y3 = y2 + h F(2.2, y2 ) and continue computing y4 , y5 , and y6 .
(d) Find approximations to y(2.2) and y(2.5).
SOLUTION
(a) With y0 = 3, t0 = 2, h = 0.1, and F(t, y) = t 2 y, we find

y1 = y0 + h F(t0 , y0 ) = 3 + 0.1(1) = 3.1.

(b) With y1 = 3.1, t1 = 2.1, h = 0.1, and F(t, y) = t 2 y, we find

y2 = y1 + h F(t1 , y1 ) = 3.1 + 0.1(4.41 3.1) = 3.231.

(c) Continuing as in the previous two parts, we find

y3 = y2 + h F(t2 , y2 ) = 3.3919;
y4 = y3 + h F(t3 , y3 ) = 3.58171;
y5 = y4 + h F(t4 , y4 ) = 3.799539;
y6 = y5 + h F(t5 , y5 ) = 4.0445851.

(d) y(2.2) y2 = 3.231, and y(2.5) y5 = 3.799539.


13. Let y(t) be the solution to y = tey satisfying y(0) = 0.
(a) Use Eulers Method with time step h = 0.1 to approximate y(0.1), y(0.2), . . . , y(1).
(b) Use separation of variables to find y(t) exactly.
(c) Compute the error in the approximations to y(0.1), y(0.5), and y(1).
SOLUTION
(a) With y0 = 0, t0 = 0, h = 0.1, and F(t, y) = tey , we compute

n tn yn
0 0 0
1 0.1 y0 + h F(t0 , y0 ) = 0
2 0.2 y1 + h F(t1 , y1 ) = 0.01
3 0.3 y2 + h F(t2 , y2 ) = 0.029801
4 0.4 y3 + h F(t3 , y3 ) = 0.058920
5 0.5 y4 + h F(t4 , y4 ) = 0.096631
6 0.6 y5 + h F(t5 , y5 ) = 0.142026
7 0.7 y6 + h F(t6 , y6 ) = 0.194082
8 0.8 y7 + h F(t7 , y7 ) = 0.251733
9 0.9 y8 + h F(t8 , y8 ) = 0.313929
10 1.0 y9 + h F(t9 , y9 ) = 0.379681

(b) Rewrite
dy
= tey as e y d y = t dt,
dt
and then integrate both sides to obtain
1 2
ey = t + C.
2
Thus,
 
1 
y = ln  t 2 + C  .
2
Applying the initial condition
 y(0) = 0 yields 0 = ln |C|, so C = 1. The exact solution to the initial value problem is
then y = ln 12 t 2 + 1 .
S E C T I O N 10.2 Graphical and Numerical Methods 1047

(c) The three errors requested are computed here:

|y(0.1) y1 | = |0.00498754 0| = 0.00498754;


|y(0.5) y5 | = |0.117783 0.0966314| = 0.021152;
|y(1) y10 | = |0.405465 0.379681| = 0.025784.

In Exercises 1419, use Eulers Method with h = 0.1 to approximate the given value of y(t).
14. y(0.5); y = y 2 , y(0) = 0
SOLUTION Let F(t, y) = y 2 . With t0 = 0, y0 = 0, and h = 0.1, we compute

y(0.0) = y0 =0
y(0.1) y1 = y0 + 0.1(y0 )2 = 0
y(0.2) y2 = y1 + 0.1(y1 )2 = 0
y(0.3) y3 = y2 + 0.1(y2 )2 = 0
y(0.4) y4 = y3 + 0.1(y3 )2 = 0
y(0.5) y5 = y4 + 0.1(y4 )2 = 0
Clearly, y(t) = 0 for all t is the exact solution to the initial value problem.
15. y(1); y = y, y(0) = 0
SOLUTION Let F(t, y) = y. With t0 = 0, y0 = 0, and h = 0.1, we compute

y(0.0) = y0 =0
y(0.1) y1 = y0 + 0.1(y0 ) =0
y(0.2) y2 = y1 + 0.1(y1 ) =0
y(0.3) y3 = y2 + 0.1(y2 ) =0
y(0.4) y4 = y3 + 0.1(y3 ) =0
y(0.5) y5 = y4 + 0.1(y4 ) =0
y(0.6) y6 = y5 + 0.1(y5 ) =0
y(0.7) y7 = y6 + 0.1(y6 ) =0
y(0.8) y8 = y7 + 0.1(y7 ) =0
y(0.9) y9 = y8 + 0.1(y8 ) =0
y(1.0) y10 = y9 + 0.1(y9 )=0

16. y(0.7); y = yt, y(0) = 1


SOLUTION Let F(t, y) = yt. With t0 = 0, y0 = 1, and h = 0.1, we compute

y(0.0) = y0 =1
y(0.1) y1 = y0 + 0.1(y0 t0 ) = 1
y(0.2) y2 = y1 + 0.1(y1 t1 ) = .99
y(0.3) y3 = y2 + 0.1(y2 t2 ) = .9702
y(0.4) y4 = y3 + 0.1(y3 t3 ) = .941094
y(0.5) y5 = y4 + 0.1(y4 t4 ) = .903450
y(0.6) y6 = y5 + 0.1(y5 t5 ) = .858278
y(0.7) y7 = y6 + 0.1(y6 t6 ) = .806781

17. y(1.5); y = t sin y, y(1) = 2


SOLUTION Let F(t, y) = t sin y. With t0 = 1, y0 = 2 and h = 0.1, we compute

y(1.0) = y0 =2
y(1.1) y0 + 0.1(t0 sin(y0 )) = 2.09093
y(1.2) y1 + 0.1(t1 sin(y1 )) = 2.18638
y(1.3) y2 + 0.1(t2 sin(y2 )) = 2.28435
y(1.4) y3 + 0.1(t3 sin(y3 )) = 2.38264
y(1.5) y4 + 0.1(t4 sin(y4 )) = 2.47898
1048 C H A P T E R 10 I N T R O D U C T I O N TO D I F F E R E N T I A L E Q U ATI O N S

18. y(2.6); y = t/y, y(0) = 2


SOLUTION Let F(t, y) = t/y. With t0 = 0, y0 = 2, and h = 0.1, we compute

y(0.0) = y0 =2
y(0.1) y1 = y0 + 0.1(t0 /y0 ) =2
y(0.2) y2 = y1 + 0.1(t1 /y1 ) = 2.005
y(0.3) y3 = y2 + 0.1(t2 /y2 ) = 2.014975
y(0.4) y4 = y3 + 0.1(t3 /y3 ) = 2.029864
y(0.5) y5 = y4 + 0.1(t4 /y4 ) = 2.049569
..
.
y(2.4) y24 = y23 + 0.1(t23 /y23 ) = 3.095655
y(2.5) y25 = y24 + 0.1(t24 /y24 ) = 3.173183
y(2.6) y26 = y25 + 0.1(t25 /y25 ) = 3.251968

19. y(0.5); y = t y, y(0) = 1


SOLUTION Let F(t, y) = t y. With t0 = 0, y0 = 1, and h = 0.1, we compute

y(0.0) = y0 =1
y(0.1) y1 = y0 + 0.1(t0 y0 ) = 0.9
y(0.2) y2 = y1 + 0.1(t1 y1 ) = 0.82
y(0.3) y3 = y2 + 0.1(t2 y2 ) = 0.758
y(0.4) y4 = y3 + 0.1(t3 y3 ) = 0.7122
y(0.5) y5 = y4 + 0.1(t4 y4 ) = 0.68098

Further Insights and Challenges


 t
20. If f (t) is continuous on [a, b], then the solution to y = f (t) with initial condition y(a) = 0 is y(t) = f (u) du.
a
ba
Show that Eulers Method with time step h = for N steps yields the N th left-endpoint approximation to
 b N
y(b) = f (u) du.
a
SOLUTION For a differential equation of the form y = f (t), the equation for Eulers method reduces to

yk = yk1 + h f (tk1 ).

With a step size of h = (b a)/N , y(b) = y N . Starting from y0 = 0, we compute

y1 = y0 + h f (t0 ) = h f (t0 )
 
y2 = y1 + h f (t1 ) = h f (t0 ) + f (t1 )
 
y3 = y2 + h f (t2 ) = h f (t0 ) + f (t1 ) + f (t2 )
..
.
  
N 1
y N = y N1 + h f (t N 1 ) = h f (t0 ) + f (t1 ) + f (t2 ) + . . . + f (t N 1 ) = h f (tk )
k=0
 b
Observe this last expression is exactly the N th left-endpoint approximation to y(b) = f (u) du.
a
In Exercises 2122, use a modification of Eulers Method, Eulers Midpoint Method, that gives a significant improvement
in accuracy. With time step h and initial value y0 = y(t0 ), the values yk are defined successively by the equation

yk = yk1 + hm k1
 
h h
where m k1 = F tk1 + , yk1 + F(tk1 , yk1 ) .
2 2
21. Apply both Eulers Method and the Euler Midpoint Method with h = 0.1 to estimate y(1.5), where y(t) satisfies
y = y with y(0) = 1. Find y(t) exactly and compute the errors in these two approximations.
S E C T I O N 10.2 Graphical and Numerical Methods 1049

SOLUTION Let F(t, y) = y. With t0 = 0, y0 = 1, and h = 0.1, fifteen iterations of Eulers method yield

y(1.5) y15 = 4.177248.

The Euler midpoint approximation with F(t, y) = y is


 
h h h
m k1 = F tk1 + , yk1 + F(tk1 , yk1 ) = yk1 + yk1
2 2 2
  2
h h
yk = yk1 + h yk1 + yk1 = yk1 + hyk1 + y
2 2 k1
Fifteen iterations of Eulers midpoint method yield:

y(1.5) y15 = 4.471304.

The exact solution to y  = y, y(0) = 1 is y(t) = et ; therefore y(1.5) = 4.481689. The error from Eulers method
is |4.177248 4.481689| = 0.304441, while the error from Eulers midpoint method is |4.471304 4.481689| =
0.010385.
 t
22. If f (t) is continuous on [a, b], then the solution to y = f (t) with initial condition y(a) = 0 is y(t) = f (u) du.
a
ba
Show that the Euler Midpoint Method with time step h = for N steps yields the N th midpoint approximation to
 b N
y(b) = f (u) du.
a
SOLUTION For a differential equation of the form y = f (t), the equations for Eulers midpoint method reduce to
   
h h
m k1 = f tk1 + and yk = yk1 + h f tk1 + .
2 2
With a step size of h = (b a)/N , y(b) = y N . Starting from y0 = 0, we compute
   
h h
y1 = y0 + h f t0 + = h f t0 +
2 2
      
h h h
y2 = y1 + h f t1 + = h f t0 + + f t1 +
2 2 2
        
h h h h
y3 = y2 + h f t2 + = h f t0 + + f t1 + + f t2 +
2 2 2 2
..
.
          
h h h h h
y N = y N1 + h f t N 1 + = h f t0 + + f t1 + + f t2 + + . . . + f t N 1 +
2 2 2 2 2
N1  
h
=h f tk +
k=0
2
 b
Observe this last expression is exactly the N th midpoint approximation to y(b) = f (u) du.
a

In Exercises 2326, use Eulers Midpoint Method with h = 0.1 to approximate the given value of y(t).

23. y(0.5); y = y 2 , y(0) = 0


SOLUTION Let F(t, y) = y 2 . With t0 = 0, y0 = 0, and h = 0.1, we compute

k tk mk yk
0 0.0 0 0
1 0.1 0 0
2 0.2 0 0
3 0.3 0 0
4 0.4 0 0
5 0.5 0 0
1050 C H A P T E R 10 I N T R O D U C T I O N TO D I F F E R E N T I A L E Q U ATI O N S

Thus, y(0.5) y5 = 0.
24. y(1); y = y 2 , y(0) = 0
SOLUTION Let F(t, y) = y 2 . With t0 = 0, y0 = 0, and h = 0.1, we compute

k tk mk yk
0 0.0 0 0
1 0.1 0 0
2 0.2 0 0
3 0.3 0 0
4 0.4 0 0
5 0.5 0 0
6 0.6 0 0
7 0.7 0 0
8 0.8 0 0
9 0.9 0 0
10 1.0 0 0

Thus, y(1.0) y10 = 0.


25. y(1); y = t + y, y(0) = 0
SOLUTION Let F(t, y) = t + y. With t0 = 0, y0 = 0, and h = 0.1, we compute

k tk mk yk
0 0.0 0 0
1 0.1 0.05 0.005
2 0.2 0.16025 0.021025
3 0.3 0.282076 0.0492326
4 0.4 0.416694 0.0909021
5 0.5 0.565447 0.147447
6 0.6 0.729819 0.220429
7 0.7 0.911450 0.311574
8 0.8 1.112152 0.422789
9 0.9 1.333928 0.556182
10 1.0 1.578991 0.714081

Hence y(1) y10 = 0.714081.


26. y(0.5); y = y 2 t, y(0) = 1
SOLUTION Let F(t, y) = y 2 t. With t0 = 0, y0 = 1, and h = 0.1, we compute

k tk mk yk
0 0.0 0 1
1 0.1 1.0525 1.10525
2 0.2 1.198685 1.225118
3 0.3 1.414524 1.366571
4 0.4 1.737871 1.540358
5 0.5 2.236298 1.763988

Therefore y(0.5) y5 = 1.763988.


S E C T I O N 10.3 The Logistic Equation 1051

10.3 The Logistic Equation


Preliminary Questions
1. Which of the following is a logistic differential equation?
 y
(a) y = 2y(1 y 2 ) (b) y = 2y 1
 3
x
(c) y = 2y 1 (d) y = 2y(1 3y)
4
SOLUTION The differential equations in (b) and (d) are logistic equations. The equation in (a) is not a logistic equation
because of the y 2 term inside the parentheses on the right-hand side; the equation in (c) is not a logistic equation because
of the presence of the independent variable on the right-hand side.
2. True or false? The logistic equation is linear.
SOLUTION False, the logistic equation is not a linear differential equation.
3. True or false? The logistic equation is separable.
SOLUTION True, the logistic equation is a separable differential equation.
4. Let y(t) be a solution to y = 4y(3 y). What is lim y(t) in the following three cases:
t
(a) y(0) = 3 (b) y(0) = 4 (c) y(0) = 2
SOLUTION
(a) If y(0) = 3, then y = 0, and y(t) = 3 for all t. Thus, lim y(t) = 3.
t
(b) If y(0) = 4, then y < 0, and lim y(t) = 3.
t
(c) If y(0) = 2, then y < 0, and lim y(t) .
t

Exercises
1. Find the general solution of the logistic equation

y = 3y(1 y/5)

Then find the particular solution satisfying y(0) = 2.



SOLUTION y = 3y(1 y/5) is a logistic equation with k = 3 and A = 5; therefore, the general solution is

5
y= .
1 e3t /C
The initial condition y(0) = 2 allows us to determine the value of C:
5 1 5 2
2= ; 1 = ; so C = .
1 1/C C 2 3
The particular solution is then
5 10
y= = .
1 + 32 e3t 2 + 3e3t

2. Find the solution of y = 2y(3 y), y(0) = 10.


SOLUTION By rewriting
 y
2y(3 y) as 6y 1 ,
3
we identify the given differential equation as a logistic equation with k = 6 and A = 3. The general solution is therefore
3
y= .
1 e6t /C
The initial condition y(0) = 10 allows us to determine the value of C:
3 1 3 10
10 = ; 1 = ; so C = .
1 1/C C 10 7
The particular solution is then
3 30
y= = .
7 e6t
1 10 10 7e6t
1052 C H A P T E R 10 I N T R O D U C T I O N TO D I F F E R E N T I A L E Q U ATI O N S

3. Let y(t) be a solution of y = 0.5y(1 0.5y) such that y(0) = 4. Determine lim y(t) without finding y(t) explicitly.
t
SOLUTION The equation is a logistic equation with k = 0.5 and A = 2. Let F(y) = 0.5y(1 0.5y), and take y0 = 4.
Then, F(y0 ) = 0.5(4)(1 0.5(4)) < 0, so, y(t) is strictly decreasing and

lim y(t) = A = 2.
t

4. Let y(t) be a solution of y = 5y(1 y/A) satisfying y(0) = 10. Find lim y(t), assuming that
t
(a) A = 15 (b) A = 5 (c) A = 10
SOLUTION Let F(y) = 5y(1 y/A).
(a) In this case, F(y0 ) = 5(10)(1 10/15) > 0, so y(t) is strictly increasing with

lim y(t) = 15.


t

(b) In this case, F(y0 ) = 5(10)(1 10/5) < 0, so y(t) is strictly decreasing with

lim y(t) = 5.
t

(c) In this case, F(y0 ) = 5(10)(1 10/10) = 0, so the solution is constant with y = 10; consequently,

lim y(t) = 10.


t

5. A population of squirrels lives in a forest with a carrying capacity of 2,000. Assume logistic growth with growth
constant k = 0.6 yr1 .
(a) Find a formula for the squirrel population P(t), assuming an initial population of 500 squirrels.
(b) How long will it take for the squirrel population to double?
SOLUTION
(a) Since k = 0.6 and the carrying capacity is A = 2000, the population P(t) of the squirrels satisfies the differential
equation

P  (t) = 0.6P(t)(1 P(t)/2000),

with general solution


2000
P(t) = .
1 e0.6t /C
The initial condition P(0) = 500 allows us to determine the value of C:
2000 1 1
500 = ; 1 = 4; so C = .
1 1/C C 3
The formula for the population is then
2000
P(t) = .
1 + 3e0.6t
(b) The squirrel population will have doubled at the time t where P(t) = 1000. This gives
2000 5
1000 = ; 1 + 3e0.6t = 2; so t = ln 3 1.83.
1 + 3e0.6t 3
It therefore takes approximately 1.83 years for the squirrel population to double.
6. The population P(t) of mosquito larvae growing in a tree hole increases according to the logistic equation with
growth constant k = 0.3 days1 and carrying capacity A = 500.
(a) Find a formula for the larvae population P(t), assuming an initial population of P0 = 50 larvae.
(b) After how many days will the larvae population reach 200?
SOLUTION
(a) Since k = 0.3 and A = 500, the population of the larvae satisfies the differential equation

P  (t) = 0.3P(t)(1 P(t)/500),

with general solution


500
P(t) = .
1 e0.3t /C
S E C T I O N 10.3 The Logistic Equation 1053

The initial condition P(0) = 50 allows us to determine the value of C:


500 1 1
50 = ; 1 = 10; so C = .
1 1/C C 9
The particular solution is then
500
P(t) = .
1 + 9e0.3t
(b) The population will reach 200 after t days, where P(t) = 200. This gives
500 10
200 = ; 1 + 9e0.3t = 2.5; so t = ln 6 5.97.
1 + 9e0.3t 3
It therefore takes approximately 5.97 days for the larvae to reach 200 in number.
7. Sunset Lake is stocked with 2,000 rainbow trout and after 1 year the population has grown to 4,500. Assuming
logistic growth with a carrying capacity of 20,000, find the growth constant k (specify the units) and determine when the
population will increase to 10,000.
SOLUTION Since A = 20000, the trout population P(t) satisfies the logistic equation

P  (t) = k P(t)(1 P(t)/20000),

with general solution


20000
P(t) = .
1 ekt /C
The initial condition P(0) = 2000 allows us to determine the value of C:
20000 1 1
2000 = ; 1 = 10; so C = .
1 1/C C 9
After one year, we know the population has grown to 4500. Lets measure time in years. Then
20000
4500 =
1 + 9ek
40
1 + 9ek =
9
31
ek =
81
81
k = ln 0.9605 years1 .
31
The population will increase to 10000 at time t where P(t) = 10000. This gives
20000
10000 =
1 + 9e0.9605t
1 + 9e0.9605t = 2
1
e0.9605t =
9
1
t= ln 9 2.29 years.
0.9605

8. Spread of a Rumor A rumor spreads through a small town. Let y(t) be the fraction of the population that has heard
the rumor at time t and assume that the rate at which the rumor spreads is proportional to the product of the fraction y of
the population that has heard the rumor and the fraction 1 y that has not yet heard the rumor.
(a) Write down the differential equation satisfied by y in terms of a proportionality factor k.
(b) Find k (in units of days1 ), assuming that 10% of the population knows the rumor at t = 0 and 40% knows it at
t = 2 days.
(c) Using the assumptions of part (b), determine when 75% of the population will know the rumor.
SOLUTION
1054 C H A P T E R 10 I N T R O D U C T I O N TO D I F F E R E N T I A L E Q U ATI O N S

(a) y  (t) is the rate at which the rumor is spreading, in percentage of the population per day. By the description given,
the rate satisfies:

y  (t) = ky(1 y),

where k is a constant of proportionality.


(b) The equation in part (a) is a logistic equation with constant k and capacity 1 (no more than 100% of the population
can hear the rumor). Thus, y takes the form
1
y(t) = .
1 ekt /C
1 allows us to determine the value of C:
The initial condition y(0) = 10

1 1 1 1
= ; 1 = 10; so C = .
10 1 1/C C 9

The condition y(2) = 25 now allows us to determine the value of k:

2 1 5 1
= ; 1 + 9e2k = ; so k = ln 6 0.896 days1 .
5 1 + 9e2k 2 2
The particular solution of the differential equation for y is then
1
y(t) = .
1 + 9e0.896t
(c) If 75% of the population knows the rumor at time t, we have
3 1
=
4 1 + 9e0.896t
4
1 + 9e0.896t =
3
ln 27
t= 3.67839
0.896
Thus, 75% of the population knows the rumor after approximately 3.67 days.
9. A rumor spreads through a school with 1,000 students. At 8 AM, 80 students have heard the rumor, and by noon, half
the school has heard it. Using the logistic model of Exercise 8, determine when 90% of the students will have heard the
rumor.
SOLUTION Let y(t) be the proportion of students that have heard the rumor at a time t hours after 8 AM. In the logistic
model of Exercise 8, we have a capacity of A = 1 (100% of students) and an unknown growth factor of k. Hence,
1
y(t) = .
1 ekt /C
The initial condition y(0) = 0.08 allows us to determine the value of C:
2 1 1 25 2
= ; 1 = ; so C = .
25 1 1/C C 2 23
so that
2
y(t) = .
2 + 23ekt
The condition y(4) = .5 now allows us to determine the value of k:
1 2 1 23
= ; 2 + 23e4k = 4; so k = ln 0.6106 hours1 .
2 2 + 23e4k 4 2
90% of the students have heard the rumor when y(t) = .9. Thus
9 2
=
10 2 + 23e0.6106t
20
2 + 23e0.6106t =
9
1 207
t= ln 7.6 hours.
0.6106 2
Thus, 90% of the students have heard the rumor after 7.6 hours, or at 3:36 PM.
S E C T I O N 10.3 The Logistic Equation 1055

10. A simpler model for the spread of a rumor assumes that the rate at which the rumor spreads is proportional
(with factor k) to the fraction of the population that has not yet heard the rumor.
(a) Compute the solutions to this model and the model of Exercise 8 with the values k = 0.9 and y0 = 0.1.
(b) Graph the two solutions on the same axis.
(c) Which model seems more realistic? Why?
SOLUTION
(a) Let y(t) denote the fraction of a population that has heard a rumor, and suppose the rumor spreads at a rate propor-
tional to the fraction of the population that has not yet heard the rumor. Then

y  = k(1 y),

for some constant of proportionality k. Separating variables and integrating both sides yields
dy
= k dt
1y
ln |1 y| = kt + C.

Thus,

y(t) = 1 Aekt ,

where A = eC is an arbitrary constant. The initial condition y(0) = 0.1 allows us to determine the value of A:

0.1 = 1 A so A = 0.9.

With k = 0.9, we have y(t) = 1 0.9e0.9t .


Using the model from Exercise 8 with k = 0.9 and y(0) = 0.1, we find
1
y(t) = .
1 + 9e0.9t
(b) The figure below shows the solutions from part (a): the solid curve corresponds to the model presented in this exercise
while the dashed curve corresponds to the model from Exercise 8.

0.8

0.6

0.4

0.2

2 4 6 8

(c) The model from Exercise 8 seems more realistic because it predicts the rumor starts spreading slowly, picks up speed
and then levels off as we near the time when the entire population has heard the rumor.
11. Let k = 1 and A = 1 in the logistic equation.
(a) Find the solutions satisfying y1 (0) = 10 and y2 (0) = 1.
(b) Find the time t when y1 (t) = 5.
(c) When does y2 (t) become infinite?
SOLUTION The general solution of the logistic equation with k = 1 and A = 1 is

1
y(t) = .
1 et /C

(a) Given y1 (0) = 10, we find C = 10


9 , and

1 10
y1 (t) = = .
1 10 e t 10 9et
9

On the other hand, given y2 (0) = 1, we find C = 12 , and

1
y2 (t) = .
1 2et
1056 C H A P T E R 10 I N T R O D U C T I O N TO D I F F E R E N T I A L E Q U ATI O N S

(b) From part (a), we have


10
y1 (t) = .
10 9et
Thus, y1 (t) = 5 when
10 9
5= ; 10 9et = 2; so t = ln .
10 9et 8
(c) From part (a), we have
1
y2 (t) = .
1 2et
Thus, y2 (t) becomes infinite when

1 2et = 0 or t = ln 2.

12. Reverse Logistic Equation Consider the logistic equation (with k, B > 0)
 
dP P
= k P 1 7
dt B
(a) Sketch the slope field of this equation.
(b) The general solution is P(t) = B/(1 ekt /C), where C is a nonzero constant. Show that P(0) > B if C > 1 and
0 < P(0) < B if C < 0.
(c) Show that Eq. (7) models an extinctionexplosion population. That is, P(t) tends to zero if the initial population
satisfies 0 < P(0) < B and it tends to after a finite amount of time if P(0) > B.
(d) Show that P = 0 is a stable equilibrium and P = B an unstable equilibrium.
SOLUTION
(a) The slope field of this equation is shown below.
y
3
2
1
0 t
1
2
3
3 2 1 0 1 2 3

 1
(b) Suppose that C > 0. Then 1 C1 < 1, 1 C1 > 1, and

B
P(0) = > B.
1 C1
 1
On the other hand, if C < 0, then 1 C1 > 1, 0 < 1 C1 < 1, and

B
0 < P(0) = < B.
1 C1

(c) From part (b), 0 < P(0) < B when C < 0. In this case, 1 ekt /C is never zero, but

ekt
1
C
as t . Thus, P(t) 0 as t . On the other hand, P(0) > B when C > 0. In this case 1 ekt /C = 0 when
t = k1 ln C. Thus,

1
P(t) as t ln C.
k
S E C T I O N 10.3 The Logistic Equation 1057

(d) Let
 
P
F(P) = k P 1 .
B

Then, F  (P) = k + 2kBP . Thus, F  (0) = k < 0, and F  (B) = k + 2k = k > 0, so P = 0 is a stable equilibrium
and P = B is an unstable equilibrium.
13. A tissue culture grows until it has a maximum area of M cm2 . The area A(t) of the culture at time t may be modeled
by the differential equation
 
A
A=k A 1 8
M
where k is a growth constant.
(a) By setting A = u 2 , show that the equation can be rewritten

2
u = 1 k 1 u
2 M

Then find the general solution using separation of variables.


(b) Show that the general solution to Eq. (8) is
2
Ce(k/ M)t 1
A(t) = M 9
Ce(k/ M)t + 1

SOLUTION
so that Eq. (8) becomes:
(a) Let A = u 2 . This gives A = 2u u,

u2
2u u = ku 1
M

2
u = k 1 u
2 M

Now, rewrite

du k u2 du
= 1 as = 12 k dt.
dt 2 M 1 u 2 /M

The partial fraction decomposition for the term on the left-hand side is
 
1 M 1 1
= + ,
1 u /M
2 2 M +u M u
so after integrating both sides, we obtain
 
M  M + u  1
ln   = kt + C.
2  M u 2

Thus,

M +u
= Ce(k/ M)t
M u

u(Ce(k/ M)t + 1) = M(Ce(k/ M)t 1)

and

Ce(k/ M)t 1
u= M .
Ce(k/ M)t + 1

(b) Recall A = u 2 . Therefore,


2
Ce(k/ M)t 1
A(t) = M .
Ce(k/ M)t + 1
1058 C H A P T E R 10 I N T R O D U C T I O N TO D I F F E R E N T I A L E Q U ATI O N S

14. Use the model of Exercise 13 to determine the area A(t) (t in hours) of a tissue culture with initial size
A(0) = 0.2 cm2 , assuming that the maximum area is M = 5 cm2 and the growth constant is k = 0.06. Graph the
solution using a graphing utility.
SOLUTION Substituting M = 5 and k = 0.06 into the result of Exercise 13(b),
2
Ce(0.06/ 5)t 1
A(t) = 5 .
Ce(0.06/ 5)t + 1
The initial condition A(0) = 0.2 gives
 
C 1 2
0.2 = 5
C +1
so C = 1.5. Hence
2
1.5e(0.06/ 5)t 1
A(t) = 5 .
1.5e(0.06/ 5)t + 1
A
5
4
3
2
1
t
50 100 150 200

15. Show that if a tissue culture grows according to Eq. (8), then the growth rate reaches a maximum when A = M/3.
A
SOLUTION According to Equation (8), the growth rate of the tissue culture is k A(1 M ). Therefore
    
d A 1 3 1 3A
k A 1 = k A1/2 k A1/2 /M = k A1/2 1 =0
dA M 2 2 2 M
when A = M/3. Because the growth rate is zero for A = 0 and for A = M and is positive for 0 < A < M, it follows
that the maximum growth rate occurs when A = M/3.
16. In 1751, Benjamin Franklin predicted that the U.S. population P(t) would increase with growth constant k =
0.028 yr1 . According to the census, the U.S. population was 5 million in 1800 and 76 million in 1900. Assuming
logistic growth with k = 0.028, what is the predicted carrying capacity for the U.S. population? Hint: Use Eqs. (3) and
(4) to show that
P(t) P0
= ekt
P(t) A P0 A

SOLUTION Assuming the population grows according to the logistic equation,

P(t)
= Cekt .
P(t) A
But
P0
C= ,
P0 A
so
P(t) P0
= ekt .
P(t) A P0 A
Now, let t = 0 correspond to the year 1800. Then the year 1900 corresponds to t = 100, and with k = 0.028, we have
76 5
= e(0.028)(100) .
76 A 5 A
Solving for A, we find

5(e2.8 1)
A= 5 943.07.
2.8 1
76 e

Thus, the predicted carrying capacity for the U.S. population is approximately 943 million.
S E C T I O N 10.3 The Logistic Equation 1059

Further Insights and Challenges


17. Let y(t) be a solution of the logistic equation
dy  y
= ky 1 10
dt A
(a) Differentiate Eq. (10) with respect to t and use the Chain Rule to show that

d2 y   
= k 2y 1 y 1
2y
dt 2 A A
(b) Show that y(t) is concave up if 0 < y < A/2 and concave down if A/2 < y < A.
(c) Show that if 0 < y(0) < A/2, then y(t) has a point of inflection at y = A/2 (Figure 5).
(d) Assume that 0 < y(0) < A/2. Find the time t when y(t) reaches the inflection point.
SOLUTION
(a) The derivative of Eq. (10) with respect to t is
      
2kyy  2y y 2y  y 2y
y  = ky  = ky  1 =k 1 ky 1 = k2 y 1 1 .
A A A A A A

(b) If 0 < y < A/2, 1 Ay and 1 2y 


A are both positive, so y > 0. Therefore, y is concave up. If A/2 < y < A,
1 Ay > 0, but 1 2y 
A < 0, so y < 0, so y is concave down.
(c) If y0 < A, y grows and lim y(t) = A. If 0 < y < A/2, y is concave up at first. Once y passes A/2, y becomes
t
concave down, so y has an inflection point at y = A/2.
(d) The general solution to Eq. (10) is
A
y= ;
1 ekt /C
thus, y = A/2 when
A A
=
2 1 ekt /C
1 ekt /C = 2
1
t = ln(C)
k
Now, C = y0 /(y0 A), so
1 y0 1 A y0
t = ln = ln .
k A y0 k y0

18. Let
A
y=
1 ekt /C
y
A

A
2
Inflection point

y(0)
t
FIGURE 5 Inflection point in a logistic curve occurs at y = A/2.

be the general nonequilibrium solution of y = ky(1 y/A) with k > 0. If y(t) has a vertical asymptote at t = tb , that
is, if lim y(t) = , we say that the solution blows up at t = tb .
ttb
(a) Show that if 0 < y(0) < A, then y does not blow up at any time tb .
(b) Show that if y(0) > A, then y blows up at a time tb , which is negative (and hence does not correspond to a real
time).
(c) Show that y blows up at some positive time tb if and only if y(0) < 0 (and hence does not correspond to a real
population).
1060 C H A P T E R 10 I N T R O D U C T I O N TO D I F F E R E N T I A L E Q U ATI O N S

SOLUTION
(a) Let y(0) = y0 . From the general solution, we find
A 1 A y0
y0 = ; 1 = ; so C = .
1 1/C C y0 y0 A

If y0 < A, then C < 0, and the denominator in the general solution, 1 ekt /C, is always positive. Thus, when
0 < y(0) < A, y does not blow up at any time.
(b) 1 ekt /C = 0 when C = ekt . Solving for t we find
1
t = ln C.
k
Because C = y yA0
and y0 > A, it follows that C > 1, and thus, ln C > 0. Therefore, y blows up at a time which is
0
negative.
(c) Suppose that y blows up at some tb > 0. From part (b), we know that
1
tb = ln C.
k
Thus, in order for tb to be positive, we must have ln C < 0, which requires C < 1. Now,
y0
C= ,
y0 A
so tb > 0 if and only if
y0 y0 A A
<1 or equivalently =1 > 1.
y0 A y0 y0
This last inequality holds if and only if y0 = y(0) < 0.

10.4 First-Order Linear Equations


Preliminary Questions
1. Which of the following are first-order linear equations?
(a) y  + x 2 y = 1 (b) y  + x y 2 = 1
(c) x 5 y  + y = e x (d) x 5 y  + y = e y
SOLUTION The equations in (a) and (c) are first-order linear differential equations. The equation in (b) is not linear
because of the y 2 factor in the second term on the left-hand side of the equation; the equation in (d) is not linear because
of the e y term on the right-hand side of the equation.
2. If (x) is an integrating factor for y  + A(x)y = B(x), then  (x) is equal to (choose the correct answer):
(a) B(x) (b) (x) A(x)

(c) (x) A (x) (d) (x)B(x)
SOLUTION The correct answer is (b): (x) A(x).

Exercises
1. Consider y  + x 1 y = x 3 .
(a) Verify that (x) = x is an integrating factor.
(b) Show that when multiplied by (x), the differential equation can be written (x y) = x 4 .
(c) Conclude that x y is an antiderivative of x 4 and use this information to find the general solution.
(d) Find the particular solution satisfying y(1) = 0.
SOLUTION
(a) The equation is of the form

y  + A(x)y = B(x)

for A(x) = x 1 and B(x) = x 3 . By Theorem 1, (x) is defined by



(x) = e A(x) d x = eln x = x.
S E C T I O N 10.4 First-Order Linear Equations 1061

(b) When multiplied by (x), the equation becomes:

x y + y = x 4.

Now, x y  + y = x y  + (x) y = (x y) , so

(x y) = x 4 .
5
(c) Since (x y) = x 4 , (x y) = x5 + C and

x4 C
y= +
5 x
(d) If y(1) = 0, we find
1 1
0= +C so = C.
5 5
The solution, therefore, is

x4 1
y= .
5 5x

dy
2. Consider + 2y = e3t .
dt
(a) Verify that (t) = e2t is an integrating factor.
(b) Use Eq. (4) to find the general solution.
(c) Find the particular solution with initial condition y(0) = 1.
SOLUTION
(a) The equation is of the form

y  + A(t)y = B(t)

for A(t) = 2 and B(t) = e3t . Thus,



(t) = e A(t) dt = e2t .

(b) According to Equation (4),


 
1
y(t) = (t)B(t) dt + C .
(t)

With (t) = e2t and B(t) = e3t , this yields


 

y(t) = e2t et dt + C = e2t C et = Ce2t e3t .

(c) Using the initial condition y(0) = 1, we find

1 = 1 + C so 2 = C.

The particular solution is therefore

y = e3t + 2e2t .

2
3. Let (x) = e x . Verify the identity

( (x)y) = (x)(y  + 2x y)

and explain how it is used to find the general solution of

y  + 2x y = x.
2
SOLUTION Let (x) = e x . Then

( (x)y) = (e x y) = 2xe x y + e x y  = e x 2x y + y  = (x) y  + 2x y .


2 2 2 2

If we now multiply both sides of the differential equation y  + 2x y = x by (x), we obtain

(x)(y  + 2x y) = x (x) = xe x .
2
1062 C H A P T E R 10 I N T R O D U C T I O N TO D I F F E R E N T I A L E Q U ATI O N S

But (x)(y  + 2x y) = ( (x)y) , so by integration we find



2 1 2
(x)y = xe x d x = e x + C.
2
Finally,
1
+ Cex .
2
y(x) =
2

4. Find the solution of y  y = e2x , y(0) = 1.


SOLUTION We first find the general solution of the differential equation y  y = e2x . This is of the standard linear
form

y  + A(x)y = B(x)

with A(x) = 1, B(x) = e2x . By Theorem 1, the integrating factor is



(x) = e A(x) d x = ex .

When multiplied by the integrating factor, the original differential equation becomes

ex y  ex y = e x or (ex y) = e x .

Integration of both sides now yields



ex y = e x d x = e x + C.

Therefore,

y(x) = e2x + Ce x .

Using the initial condition y(0) = 1, we find

1 = 1+C so 0 = C.

Therefore,

y = e2x .

In Exercises 518, find the general solution of the first-order linear differential equation.

5. x y  + y = x
SOLUTION Rewrite the equation as

1
y + y = 1,
x

which is in standard linear form with A(x) = 1x and B(x) = 1. By Theorem 1, the integrating factor is

(x) = e A(x) d x = eln x = x.

When multiplied by the integrating factor, the rewritten differential equation becomes

x y + y = x or (x y) = x.

Integration of both sides now yields


1 2
xy = x + C.
2
Finally,
1 C
y(x) = x+ .
2 x

6. x y  y = x 2 x
S E C T I O N 10.4 First-Order Linear Equations 1063

SOLUTION Rewrite the equation as

1
y y = x 1,
x

which is in standard linear form with A(x) = 1x and B(x) = x 1. By Theorem 1, the integrating factor is

(x) = e A(x) d x = e ln x = x 1 .

When multiplied by the integrating factor, the rewritten differential equation becomes
1  1 1  y  1
y 2y =1 or =1 .
x x x x x
Integration of both sides now yields
y
= x ln x + C.
x
Finally,

y(x) = x 2 x ln x + C x.

7. 3x y  y = x 1
SOLUTION Rewrite the equation as

1 1
y y = 2,
3x 3x

which is in standard form with A(x) = 13 x 1 and B(x) = 13 x 2 . By Theorem 1, the integrating factor is

(x) = e A(x) d x = e(1/3) ln x = x 1/3 .

When multiplied by the integrating factor, the rewritten differential equation becomes
1 4/3 1 1 7/3
x 1/3 y  x = x 7/3 or (x 1/3 y) = x .
3 3 3
Integration of both sides now yields
1
x 1/3 y = x 4/3 + C.
4
Finally,
1
y(x) = x 1 + C x 1/3 .
4

8. y  + x y = x
SOLUTION This equation is in standard form with A(x) = x and B(x) = x. By Theorem 1, the integrating factor is

(x) = e x d x = e(1/2)x .
2

When multiplied by the integrating factor, the original differential equation becomes

e(1/2)x y  + xe(1/2)x y = xe(1/2)x (e(1/2)x y) = xe(1/2)x .


2 2 2 2 2
or

Integration of both sides now yields

e(1/2)x y = e(1/2)x + C.
2 2

Finally,

y(x) = 1 + Ce(1/2)x .
2

9. y  + 3x 1 y = x + x 1
1064 C H A P T E R 10 I N T R O D U C T I O N TO D I F F E R E N T I A L E Q U ATI O N S

SOLUTION This equation is in standard form with A(x) = 3x 1 and B(x) = x + x 1 . By Theorem 1, the integrating
factor is

1
(x) = e 3x = e3 ln x = x 3 .

When multiplied by the integrating factor, the original differential equation becomes

x 3 y  + 3x 2 y = x 4 + x 2 or (x 3 y) = x 4 + x 3 .

Integration of both sides now yields


1 5 1 3
x3 y = x + x + C.
5 3
Finally,
1 2 1
y(x) = x + + C x 3 .
5 3

10. y  + x 1 y = cos(x 2 )
SOLUTION This equation is in standard form with A(x) = x 1 and B(x) = cos(x 2 ). By Theorem 1, the integrating
factor is

1
(x) = e x d x = eln x = x.

When multiplied by the integrating factor, the original differential equation becomes

x y  + y = x cos(x 2 ) or (x y) = x cos(x 2 ).

Integration of both sides now yields


1
xy = sin(x 2 ) + C.
2
Finally,
1 1
y(x) = x sin(x 2 ) + C x 1 .
2

11. x y  = y x
SOLUTION Rewrite the equation as

1
y y = 1,
x

which is in standard form with A(x) = 1x and B(x) = 1. By Theorem 1, the integrating factor is

(x) = e (1/x) d x = e ln x = x 1 .

When multiplied by the integrating factor, the rewritten differential equation becomes
 
1  1 1 1 1
y 2y = or y = .
x x x x x
Integration on both sides now yields
1
y = ln x + C.
x
Finally,

y(x) = x ln x + C x.

3y
12. x y  = x 3
x
S E C T I O N 10.4 First-Order Linear Equations 1065

SOLUTION Rewrite the equation as

3
y + 2 y = x 2,
x

which is in standard form with A(x) = 3x 2 and B(x) = x 2 . By Theorem 1, the integrating factor is
2
(x) = e 3x d x = e3/x .

When multiplied by the integrating factor, the rewritten differential equation becomes
3
e3/x y  + 2 e3/x y = x 2 e3/x or (e3/x y) = x 2 e3/x .
x
Integration on both sides now yields

e3/x y = x 2 e3/x d x + C.

Finally,

y(x) = e3/x x 2 e3/x d x + Ce3/x .

13. y  + y = e x
SOLUTION This equation is in standard form with A(x) = 1 and B(x) = e x . By Theorem 1, the integrating factor is

(x) = e 1 d x = e x .

When multiplied by the integrating factor, the original differential equation becomes

e x y  + e x y = e2x or (e x y) = e2x .

Integration on both sides now yields


1 2x
ex y = e + C.
2
Finally,
1 x
y(x) = e + Cex .
2

14. y  + (sec x)y = cos x


SOLUTION This equation is in standard form with A(x) = sec x and B(x) = cos x. By Theorem 1, the integrating
factor is

(x) = e sec x d x = eln(sec x+tan x) = sec x + tan x.

When multiplied by the integrating factor, the original differential equation becomes

(sec x + tan x)y  + (sec2 x + sec x tan x)y = 1 + sin x

or

((sec x + tan x)y) = 1 + sin x.

Integration on both sides now yields

(sec x + tan x)y = x cos x + C.

Finally,
x cos x + C
y(x) = .
sec x + tan x

15. y  + (tan x)y = cos x


1066 C H A P T E R 10 I N T R O D U C T I O N TO D I F F E R E N T I A L E Q U ATI O N S

SOLUTION This equation is in standard form with A(x) = tan x and B(x) = cos x. By Theorem 1, the integrating
factor is

(x) = e tan x d x = eln sec x = sec x.

When multiplied by the integrating factor, the original differential equation becomes

sec x y  + sec x tan x y = 1 or (y sec x) = 1.

Integration on both sides now yields

y sec x = x + C.

Finally,

y(x) = x cos x + C cos x.

16. e2x y  = 1 e x y
SOLUTION Rewrite the equation as

y  + ex y = e2x ,

which is in standard form with A(x) = ex and B(x) = e2x . By Theorem 1, the integrating factor is
x
 x
(x) = e e d x = ee .

When multiplied by the integrating factor, the rewritten differential equation becomes
x x x x x
ee y  + exe y = e2x ee or (ee y) = e2x ee .

Integration on both sides now yields



x x
(ee y) = e2x ee d x.

To handle the remaining integral, make the substitution u = ex , du = ex d x. Then


 
x x x
e2x ee d x = ueu du = ueu + eu + C = ex ee + ee + C.

Finally,
x
y(x) = 1 + ex + Cee .

17. y  (ln x)y = x x


SOLUTION This equation is in standard form with A(x) = ln x and B(x) = x x . By Theorem 1, the integrating factor
is
 ex
(x) = e ln x d x = e xx ln x = x .
x
When multiplied by the integrating factor, the original differential equation becomes

x x e x y  (ln x)x x e x y = e x or (x x e x y) = e x .

Integration on both sides now yields

x x e x y = e x + C.

Finally,

y(x) = x x + C x x ex .

18. y  + y = cos x
S E C T I O N 10.4 First-Order Linear Equations 1067

SOLUTION This equation is in standard form with A(x) = 1 and B(x) = cos x. By Theorem 1, the integrating factor
is

(x) = e 1 d x = e x .

When multiplied by the integrating factor, the original differential equation becomes

e x y  + e x y = e x cos x or (e x y) = e x cos x.

Integration on both sides (integration by parts is needed on the right-hand side of the equation) now yields
1 x
ex y = e (sin x + cos x) + C.
2
Finally,
1
y(x) = (sin x + cos x) + Cex .
2

In Exercises 1926, solve the initial value problem.

19. y  + 3y = e2x , y(0) = 1


SOLUTION First, we find the general solution of the differential equation. This linear equation is in standard form with
A(x) = 3 and B(x) = e2x . By Theorem 1, the integrating factor is

(x) = e3x .

When multiplied by the integrating factor, the original differential equation becomes

(e3x y) = e5x .

Integration on both sides now yields


1 5x
(e3x y) = e + C;
5
hence,
1 2x
y(x) = e + Ce3x .
5
The initial condition y(0) = 1 allows us to determine the value of C:
1 6
1 = +C so C = .
5 5
The solution to the initial value problem is therefore
1 2x 6 3x
y(x) = e e .
5 5

20. x y  + y = e x , y(1) = 3
SOLUTION First, we find the general solution of the differential equation. Rewrite the equation as

1 1
y + y = ex ,
x x
which is in standard form with A(x) = x 1 and B(x) = x 1 e x . By Theorem 1, the integrating factor is

1
(x) = e x d x = eln x = x.

When multiplied by the integrating factor, the rewritten differential equation becomes

(x y) = e x .

Integration on both sides now yields

x y = e x + C;

hence,
1 x C
y(x) = e + .
x x
1068 C H A P T E R 10 I N T R O D U C T I O N TO D I F F E R E N T I A L E Q U ATI O N S

The initial condition y(1) = 3 allows us to determine the value of C:


C
3=e+ so C = 3 e.
1
The solution to the initial value problem is therefore
1 x 3e
y(x) = e + .
x x

1
21. y  + y = x 2 , y(1) = 2
x +1
SOLUTION First, we find the general solution of the differential equation. This linear equation is in standard form with
1 and B(x) = x 2 . By Theorem 1, the integrating factor is
A(x) = x+1

(x) = e 1/(x+1) d x = eln(x+1) = x + 1.

When multiplied by the integrating factor, the original differential equation becomes

((x + 1)y) = x 1 + x 2 .

Integration on both sides now yields

(x + 1)y = ln x x 1 + C;

hence,
 
1 1
y(x) = C + ln x .
x +1 x
The initial condition y(1) = 2 allows us to determine the value of C:
1
2= (C 1) so C = 5.
2
The solution to the initial value problem is therefore
 
1 1
y(x) = 5 + ln x .
x +1 x

22. y  + y = sin x, y(0) = 1


SOLUTION First, we find the general solution of the differential equation. This equation is in standard form with
A(x) = 1 and B(x) = sin x. By Theorem 1, the integrating factor is

(x) = e 1 d x = e x .

When multiplied by the integrating factor, the original differential equation becomes

(e x y) = e x sin x.

Integration on both sides (integration by parts is needed on the right-hand side of the equation) now yields
1 x
(e x y) = e (sin x cos x) + C;
2
hence,
1
y(x) = (sin x cos x) + Cex .
2
The initial condition y(0) = 1 allows us to determine the value of C:
1 3
1= +C so C= .
2 2
The solution to the initial value problem is therefore
1 3
y(x) = (sin x cos x) + ex .
2 2

23. (sin x)y  = (cos x)y + 1, y( 4 ) = 0


S E C T I O N 10.4 First-Order Linear Equations 1069

SOLUTION First, we find the general solution of the differential equation. Rewrite the equation as

y  (cot x)y = csc x,

which is in standard form with A(x) = cot x and B(x) = csc x. By Theorem 1, the integrating factor is

(x) = e cot x d x = e ln sin x = csc x.

When multiplied by the integrating factor, the rewritten differential equation becomes

(csc x y) = csc2 x.

Integration on both sides now yields

(csc x)y = cot x + C;

hence,

y(x) = cos x + C sin x.

The initial condition y( /4) = 0 allows us to determine the value of C:



2 2
0= +C so C = 1.
2 2
The solution to the initial value problem is therefore

y(x) = cos x + sin x.


24. y  + (sec t)y = sec t,y(0) = 1 and y 4 = 1


SOLUTION First, we find the general solution of the differential equation. This equation is in standard form with
A(t) = sec t and B(t) = sec t. By Theorem 1, the integrating factor is

(t) = e sec t dt = eln(sec t+tan t) = sec t + tan t.

When multiplied by the integrating factor, the original differential equation becomes

((sec t + tan t)y) = sec2 t + sec t tan t.

Integration on both sides now yields

(sec t + tan t)y = tan t + sec t + C;

hence,
C
y(t) = 1 + .
sec t + tan t
The initial condition y(0) = 1 allows us to determine the value of C:
C
1=1+ so C = 0.
1+0
The solution to the initial value problem is therefore

y(x) = 1.

We find the same particular solution with the initial condition y( /4) = 1.
25. y  + (tanh x)y = 1, y(0) = 3
SOLUTION First, we find the general solution of the differential equation. This equation is in standard form with
A(x) = tanh x and B(x) = 1. By Theorem 1, the integrating factor is

(x) = e tanh x d x = eln cosh x = cosh x.

When multiplied by the integrating factor, the original differential equation becomes

(cosh x y) = cosh x.

Integration on both sides now yields

(cosh x y) = sinh x + C;
1070 C H A P T E R 10 I N T R O D U C T I O N TO D I F F E R E N T I A L E Q U ATI O N S

hence,

y(x) = tanh x + C sech x.

The initial condition y(0) = 3 allows us to determine the value of C:

3 = C.

The solution to the initial value problem is therefore

y(x) = tanh x + 3 sech x.

x 1
26. y  + y= , y(1) = 0
1 + x2 (1 + x 2 )3/2
SOLUTION First, we find the general solution of the differential equation. This equation is in standard form with
A(x) = x and B(x) = 1 . By Theorem 1, the integrating factor is
1+x 2 (1+x 2 )3/2
 
(x) = e (x/(1+x )) d x = e(1/2) ln(1+x ) = 1 + x 2 .
2 2

When multiplied by the integrating factor, the original differential equation becomes
  1
1 + x2 y = .
1 + x2
Integration on both sides now yields

1 + x 2 y = tan1 x + C;

hence,

tan1 x C
y(x) =  + .
1+x 2 1 + x2
The initial condition y(1) = 0 allows us to determine the value of C:
1  
0= +C so C = .
2 4 4
The solution to the initial value problem is therefore
1  
y(x) =  tan1 x .
1 + x2 4

27. Find the general solution of y  + ny = emx for all m, n. Note: The case m = n must be treated separately.
SOLUTION For any m, n, Theorem 1 gives us the formula for (x):

(x) = e n d x = enx .

When multiplied by the integrating factor, the original differential equation becomes

(enx y) = e(m+n)x .

If m
= n, integration on both sides yields
1
enx y = e(m+n)x + C,
m+n
so
1
y(x) = emx + Cenx .
m+n
However, if m = n, then m + n = 0 and the equation reduces to

(enx y) = 1,

so integration yields

enx y = x + C or y(x) = (x + C)enx .


S E C T I O N 10.4 First-Order Linear Equations 1071

28. A 200-gal tank contains 100 gal of water with a salt concentration of 0.1 lb/gal. Water with a salt concentration of
0.4 lb/gal flows into the tank at a rate of 20 gal/min. The fluid is mixed instantaneously, and water is pumped out at a
rate of 10 gal/min. Let y(t) be the amount of salt in the tank at time t.
(a) Set up and solve the differential equation for y(t).
(b) What is the salt concentration when the tank overflows?
SOLUTION Because water flows into the tank at the rate of 20 gal/min but flows out at the rate of 10 gal/min, there is
a net inflow of 10 gal/min. Therefore, at any time t, there are 100 + 10t gallons of water in the tank.
(a) The net flow of salt into the tank at time t is
     
dy gal lb gal y lb 1
= salt rate in salt rate out = 20 0.4 10 =8 y.
dt min gal min 100 + 10t gal 10 + t
Rewriting this linear equation in standard form, we have
dy 1
+ y = 8,
dt 10 + t
1 and B(t) = 8. By Theorem 1, the integrating factor is
so A(t) = 10+t
 1
(t) = e (10+t) dt = eln(10+t) = 10 + t.

When multiplied by the integrating factor, the rewritten differential equation becomes

((10 + t)y) = 80 + 8t.

Integration on both sides now yields

(10 + t)y = 80t + 4t 2 + C;

hence,

80t + 4t 2 + C
y(t) = .
10 + t
The initial condition y(0) = 10 allows us to determine the value of C:
C
10 = so C = 100.
10
The solution to the initial value problem is therefore

80t + 4t 2 + 100
y(t) = .
10 + t
(b) The tank overflows when t = 10. The amount of salt in the tank at that time is
800 + 400 + 100
y(10) = = 65 lb,
20
so the concentration of salt is
65 lb
= 0.325 lb/gal.
200 gal

29. Repeat Exercise 28(a), assuming that water is pumped out at a rate of 20 gal/min. What is the limiting salt concen-
tration for large t?
SOLUTION Because water flows into the tank at the same rate as water flows out of the tank, the amount of water in
the tank remains a constant 100 gallons. Now, let y(t) be the amount of salt in the tank (in pounds) at any time t (in
minutes). The net flow of salt into the tank t is
     
dy gal lb gal y lb 1
= salt rate in salt rate out = 20 0.4 20 = 8 y.
dt min gal min 100 gal 5
Rewriting this linear equation in standard form, we have
dy 1
+ y = 8,
dt 5
so A(t) = 15 and B(t) = 8. By Theorem 1, the integrating factor is

(t) = e (1/5) dt = et/5 .
1072 C H A P T E R 10 I N T R O D U C T I O N TO D I F F E R E N T I A L E Q U ATI O N S

When multiplied by the integrating factor, the rewritten differential equation becomes

(et/5 y) = 8et/5 .

Integration on both sides now yields

et/5 y = 40et/5 + C;

hence,

y(t) = 40 + Cet/5 .

The initial condition y(0) = 10 allows us to determine the value of C:

10 = 40 + C so C = 30.

The solution to the initial value problem is therefore

y(t) = 40 30et/5 .

From here, we see that y(t) 40 as t , so that the limiting concentration of salt in the tank is 100 40 = 0.4 pounds
per gallon. This is completely intuitive; as time goes on, the concentration in the tank should resemble the concentration
pouring into it.
30. Repeat Exercise 28(a), assuming that water is pumped out at a rate of 25 gal/min. What is the limiting salt concen-
tration for t large?
SOLUTION Because water flows into the tank at the rate of 20 gal/min but flows out at the rate of 25 gal/min, there is
a net outflow of 5 gal/min. Therefore, at any time t, there are 100 5t gallons of water in the tank. The net flow of salt
into the tank t is then
     
dy gal lb gal y lb 5
= salt rate in salt rate out = 20 0.4 25 =8 y.
dt min gal min 100 5t gal 20 t
Rewriting this linear equation in standard form, we have
dy 5
+ y = 8,
dt 20 t
5 and B(t) = 8. By Theorem 1, the integrating factor is
so A(t) = 20t
 1
(t) = e5 (20t) dt = e5 ln(20t) = (20 t)5 .

When multiplied by the integrating factor, the rewritten differential equation becomes

((20 t)5 y) = 8(20 t)5 .

Integration on both sides now yields

(20 t)5 y = 2(20 t)4 + C;

hence,

y(t) = 2(20 t) + C(20 t)5 .

The initial condition y(0) = 10 allows us to determine the value of C:


3
10 = 40 + C(20)5 so C = .
320000
The amount of the salt is then given by
3
y(t) = 2(20 t) (20 t)5 lb.
320000
The concentration of salt in the tank is therefore
y(t) 2 3
= (20 t)4 lb/gal.
100 5t 5 1600000
From here we see that, just before the tank empties at t = 20 minutes, the concentration of salt in the tank is 0.4 pounds
per gallon.
S E C T I O N 10.4 First-Order Linear Equations 1073

20
31. Water flows into a tank at the variable rate Rin = gal/min and out at the constant rate Rout = 5 gal/min. Let
1+t
V (t) be the volume of water in the tank at time t.
(a) Set up a differential equation for V (t) and solve it with the initial condition V (0) = 100.
(b) Find the maximum value of V .
(c) Plot V (t) and estimate the time t when the tank is empty.
SOLUTION
(a) The rate of change of the volume of water in the tank is given by
dV 20
= Rin Rout = 5.
dt 1+t
Because the right-hand side depends only on the independent variable t, we integrate to obtain

V (t) = 20 ln(1 + t) 5t + C.

The initial condition V (0) = 100 allows us to determine the value of C:

100 = 20 ln 1 0 + C so C = 100.

Therefore

V (t) = 20 ln(1 + t) 5t + 100.

(b) Using the result from part (a),


dV 20
= 5=0
dt 1+t

when t = 3. Because ddtV > 0 for t < 3 and ddtV < 0 for t > 3, it follows that

V (3) = 20 ln 4 15 + 100 112.726 gal

is the maximum volume.


(c) V (t) is plotted in the figure below at the left. On the right, we zoom in near the location where the curve crosses the
t-axis. From this graph, we estimate that the tank is empty after roughly 34.25 minutes.

120
100
80
60
40 32 34 36 38
20

10 20 30 40

32. A stream feeds into a lake at a rate of 1,000 m3 /day. The stream is polluted with a toxin whose concentration is
5 g/m3 . Assume that the lake has volume 106 m3 and that water flows out of the lake at the same rate of 1,000 m3 /day.
Set up a differential equation for the concentration c(t) of toxin in the lake and solve for c(t), assuming that c(0) = 0.
What is the limiting concentration for large t?
SOLUTION Let M(t) denote the amount of toxin, in grams, in the lake at time t. The rate at which toxin enters the lake
is given by

g m3 g
5 3 1000 = 5000 ,
m day day
while the rate at which toxin exits the lake is given by

M(t) g m3 M(t) g
1000 = ,
106 m3 day 1000 day
where we have assumed that any toxin in the lake is spread uniformly throughout the lake. The differential equation for
M(t) is then
dM M
= 5000 .
dt 1000
1074 C H A P T E R 10 I N T R O D U C T I O N TO D I F F E R E N T I A L E Q U ATI O N S

M(t)
The concentration of the toxin in the lake is given by c(t) = , so c (t) = 16 M  (t), giving
106 10

dc 1 1
= c.
dt 200 1000
Rewriting this linear equation in standard form, we have
dc 1 1
+ c= ,
dt 1000 200
1 and B(t) = 1 . By Theorem 1, the integrating factor is
so A(t) = 1000 200

(t) = et/1000 .

When multiplied by the integrating factor, the rewritten differential equation becomes
1 t/1000
(et/1000 c) = e .
200
Integration on both sides now yields

et/1000 c = 5et/1000 + A;

hence,

c(t) = 5 + Aet/1000 .

The initial condition c(0) = 0 allows us to determine the value of A:

0=5+ A so A = 5.

Therefore
 
c(t) = 5 1 et/1000 grams/m3 .

grams
As t , c(t) 5, so the limiting concentration of pollution is 5 .
m3
In Exercises 3335, consider a series circuit (Figure 4) consisting of a resistor of R ohms, an inductor of L henries, and
variable voltage source of V (t) volts (time t in seconds). The current through the circuit I (t) (in amperes) satisfies the
differential equation:
dI R 1
+ I = V (t) 12
dt L L

V(t) L

FIGURE 4 RL circuit.

33. Find the solution to Eq. (12) with initial condition I (0) = 0, assuming that R = 100 , L = 5 H, and V (t) is
constant with V (t) = 10 V.
SOLUTION If R = 100, V (t) = 10, and L = 5, the differential equation becomes

dI
+ 20I = 2,
dt

which is a linear equation in standard form with A(t) = 20 and B(t) = 2. The integrating factor is (t) = e20t , and
when multiplied by the integrating factor, the differential equation becomes

(e20t I ) = 2e20t .

Integration of both sides now yields


1 20t
e20t I = e + C;
10
S E C T I O N 10.4 First-Order Linear Equations 1075

hence,
1
I (t) = + Ce20t .
10
The initial condition I (0) = 0 allows us to determine the value of C:
1 1
0= +C so C = .
10 10
Finally,
1  
I (t) = 1 e20t .
10

34. Assume that R = 110 , L = 10 H, and V (t) = et .


(a) Solve Eq. (12) with initial condition I (0) = 0.
(b) Use a computer algebra system to sketch the graph of the solution for 0 t 3.
(c) Calculate tm and I (tm ), where tm is the time at which I (t) has a maximum value.
SOLUTION
(a) If R = 110, V (t) = et , and L = 10, the differential equation becomes
dI 1 t
+ 11I = e ,
dt 10
which is a linear equation in standard form with A(t) = 11 and B(t) = 10 1 et . The integrating factor is (t) = e11t ,
and when multiplied by the integrating factor, the differential equation becomes
1 10t
(e11t I ) = e .
10
Integration of both sides now yields
1 10t
e11t I = e + C;
100
hence,
1 t
I (t) = e + Ce11t .
100
The initial condition I (0) = 0 allows us to determine the value of C:
1 1
0= +C so C = .
100 100
Finally,
1  t 
I (t) = e e11t .
100
(b) The graph of I (t) is shown below.
I

0.006
0.004
0.002
t
0 0.5 1 1.5 2 2.5 3

(c) Using the result from part (a),


dI 1  t 
= e + 11e11t = 0
dt 100
when
1
t = tm = ln 11 seconds.
10
Now,
1  (1/10) ln 11  1  1/10 
I (tm ) = e e(11/10) ln 11 = 11 1111/10 0.00715 amperes.
100 100
1076 C H A P T E R 10 I N T R O D U C T I O N TO D I F F E R E N T I A L E Q U ATI O N S

35. Assume that V (t) = V is constant and I (0) = 0.


(a) Solve for I (t).
(b) Show that lim I (t) = V /R and that I (t) reaches approximately 63% of its limiting value after L/R seconds.
t
(c) How long does it take for I (t) to reach 90% of its limiting value if R = 500 , L = 4 H, and V = 20 V?
SOLUTION
(a) The equation
dI R 1
+ I = V
dt L L

is a linear equation in standard form with A(t) = LR and B(t) = L1 V (t). By Theorem 1, the integrating factor is

(t) = e (R/L) dt = e(R/L) t .

When multiplied by the integrating factor, the original differential equation becomes
V
(e(R/L) t I ) = e(R/L) t .
L
Integration on both sides now yields
V (R/L) t
(e(R/L) t I ) = e + C;
R
hence,
V
I (t) = + Ce(R/L) t .
R
The initial condition I (0) = 0 allows us to determine the value of C:
V V
0= +C so C = .
R R
Therefore the current is given by
V  
I (t) = 1 e(R/L) t .
R

(b) As t , e(R/L) t 0, so I (t) VR . Moreover, when t = (L/R) seconds, we have


 
L V   V   V
I = 1 e(R/L) (L/R) = 1 e1 0.632 .
R R R R
(c) Using the results from part (a) and part (b), I (t) reaches 90% of its limiting value when
9
= 1 e(R/L) t ,
10
or when
L
t= ln 10.
R
With L = 4 and R = 500, this takes approximately 0.0184 seconds.

36. Tank 1 in Figure 5 is filled with V1 gallons of water. Water flows into the tank at a rate of R gal/min and
out through the bottom at the same rate R. Suppose that I gallons of blue ink are dumped into the tank at time t = 0 and
mixed instantaneously. Let y1 (t) be the quantity of ink in the tank at time t.
dy R
(a) Explain why y1 satisfies the differential equation 1 = y1 .
dt V1
(b) Solve for y1 (t) with V1 = 100, R = 10, and I = 2.
SOLUTION
(a) After the initial I gallons of blue ink are dumped into Tank 1, no additional blue ink is added. The rate at which blue
ink enters the tank is therefore zero. The rate at which ink leaves the tank is the product of the rate R at which water
leaves the tank and the concentration y1 /V1 of the ink in the water. Thus,
d y1 R
= y1 .
dt V1
S E C T I O N 10.4 First-Order Linear Equations 1077

(b) If V1 = 100 and R = 10, our equation becomes


d y1 1 dy 1
= y1 or + y = 0.
dt 10 dt 10 1
The integrating factor for this equation is (t) = et/10 . When multiplied by the integrating factor, the differential
equation becomes

(et/10 y1 ) = 0.

Integration yields

et/10 y1 = C or y1 (t) = Cet/10 .

The initial condition y1 (0) = 2 allows us to determine the value of C: C = 2. Finally,

y1 (t) = 2et/10 .

37. Continuing with the previous exercise, let Tank 2 be another tank filled with V2 gallons of water. Assume
that the inky water from Tank 1 empties into Tank 2 as in Figure 5, mixes instantaneously, and leaves Tank 2 at the same
rate R. Let y2 (t) be the amount of ink in Tank 2 at time t.

R (gal/min)

Tank 1

R (gal/min)

Tank 2

R (gal/min)

FIGURE 5

(a) Explain why y2 satisfies the differential equation


 
d y2 y1 y
=R 2
dt V1 V2
(b) Use the solution to Exercise 36 to solve for y2 (t) if V1 = 100, V2 = 200, R = 10, I = 2, and y2 (0) = 0.
(c) Plot the solution for 0 t 120.
(d) Find the maximum ink concentration in Tank 2.
SOLUTION
(a) The water flowing into Tank 2 has an ink concentration of y1 (t)/V1 and the water flowing out from the tank has an
ink concentration of y2 (t)/V2 . Thus,
 
d y2 y1 y
=R 2 .
dt V1 V2

(b) With V1 = 100, R = 10 and I = 2, we know from the previous exercise that y1 (t) = 2et/10 . Substituting this
expression and the given parameter values into the differential equation obtained in part (a), we have
 
d y2 2 t/10 1 1 1
= 10 e y2 = et/10 y .
dt 100 200 5 20 2
Hence,
d y2 1 1
+ y2 = et/10 .
dt 20 5
The integrating factor for this linear equation is (t) = et/20 so we get
  1
et/20 y2 = et/20 ,
5
and

et/20 y2 = 4et/20 + C.
1078 C H A P T E R 10 I N T R O D U C T I O N TO D I F F E R E N T I A L E Q U ATI O N S

Thus,

y2 (t) = 4et/10 + Cet/20 .

The initial condition y2 (0) = 0 allows us to determine C = 4; consequently,


 
y2 (t) = 4 et/20 et/10 .

(c) A plot of y2 (t) is shown below.


y
1.0
0.8
0.6
0.4
0.2
t
0 20 40 60 80 100 120

(d) Using the result of part (b), we find that


 
d y2 1 t/10 1 t/20
=4 e e =0
dt 10 20
when

t = 20 ln 2.

Thus, the maximum ink concentration in Tank 2 is


 
1 1
y2 (20 ln 2) = 4 = 1.
2 4

Further Insights and Challenges


38. Let (x) be an integrating factor for y  + A(x)y = B(x). The differential equation y  + A(x)y = 0 is called the
associated homogeneous equation.
1
(a) Show that is a solution of the associated homogeneous equation.
(x)
C
(b) Show that if y = f (x) is a particular solution of y  + A(x)y = B(x), then f (x) + is also a solution for any
(x)
constant C.
SOLUTION

(a) Remember that  (x) = A(x) (x). Now, let y(x) = ( (x))1 . Then
1 A(x) 1 A(x)
y  + A(x)y =  (x) + = A(x) (x) + = 0.
( (x))2 (x) ( (x)) 2 (x)

(b) Suppose f (x) satisfies f  (x) + A(x) f (x) = B(x). Now, let y(x) = f (x) + C/ (x), where C is an arbitrary
constant. Then
C C A(x)
y  + A(x)y = f  (x)  (x) + A(x) f (x) +
( (x))2 (x)
 

C  (x)
= f  (x) + A(x) f (x) + A(x) = B(x) + 0 = B(x).
(x) (x)

39. Use the Fundamental Theorem of Calculus and the Product Rule to verify directly that for any x0 , the function
 x
f (x) = (x)1 (t)B(t) dt
x0

is a solution of the initial value problem

y  + A(x)y = B(x), y(x 0 ) = 0,

where (x) is an integrating factor [a solution to Eq. (3)].


S E C T I O N 10.4 First-Order Linear Equations 1079

SOLUTION Remember that  (x) = A(x) (x). Now, let


 x
1
y(x) = (t)B(t) dt.
(x) x0
Then,
 x0
1
y(x 0 ) = (t)B(t) dt = 0,
(x) x0
and
 x 
 (x) A(x) x
y  + A(x)y = (t)B(t) dt + B(x) + (t)B(t) dt
( (x))2 x0 (x) x0
  x
A(x) A(x)
= B(x) + + (t)B(t) dt = B(x).
(x) (x) x0

40. Transient Currents Suppose the circuit described by Eq. (12) is driven by a sinusoidal voltage source V (t) =
V sin t (where V and are constant).
(a) Show that
V
I (t) = (R sin t L cos t) + Ce(R/L) t
R2 + L 2 2

(b) Let Z = R 2 + L 2 2 . Choose so that Z cos = R and Z sin = L . Use the addition formula for the sine
function to show that
V
I (t) = sin( t ) + Ce(R/L) t
Z
This shows that the current in the circuit varies sinusoidally apart from a DC term (called the transient current in
electronics) that decreases exponentially.
SOLUTION
(a) With V (t) = V sin t, the equation
dI R 1
+ I = V (t)
dt L L
becomes
dI R V
+ I = sin t.
dt L L
This is a linear equation in standard form with A(t) = LR and B(t) = VL sin t. By Theorem 1, the integrating factor is
 
(t) = e A(t) dt = e(R/L) t .

When multiplied by the integrating factor, the equation becomes


V (R/L) t
(e(R/L) t I ) = e sin t.
L
Integration on both sides (integration by parts is needed for the integral on the right-hand side) now yields
V
(e(R/L) t I ) = e(R/L) t (R sin t L cos t) + C;
R2 + L 2 2
hence,
V
I (t) = (R sin t L cos t) + Ce(R/L) t .
R2 + L 2 2

(b) Let Z = R 2 + L 2 2 , and choose so that Z cos = R and Z sin = L . Then
V V
(R sin t L cos t) = (Z cos sin t Z sin cos t)
R2 + L 2 2 Z2
V V
= (cos sin t sin cos t) = sin( t ).
Z Z
Thus,
V
I (t) = sin( t ) + Ce(R/L) t .
Z
1080 C H A P T E R 10 I N T R O D U C T I O N TO D I F F E R E N T I A L E Q U ATI O N S

CHAPTER REVIEW EXERCISES


1. Which of the following differential equations are linear? Determine the order of each equation.
(a) y  = y 5 3x 4 y (b) y  = x 5 3x 4 y

(c) y = y  3x y (d) sin x y  = y 1
SOLUTION

(a) y 5 is a nonlinear term involving the dependent variable, so this is not a linear equation; the highest order derivative
that appears in the equation is a first derivative, so this is a first-order equation.
(b) This is linear equation; the highest order derivative that appears in the equation is a first derivative, so this is a
first-order equation.

(c) y is a nonlinear term involving the dependent variable, so this is not a linear equation; the highest order derivative
that appears in the equation is a third derivative, so this is a third-order equation.
(d) This is linear equation; the highest order derivative that appears in the equation is a second derivative, so this is a
second-order equation.
2. Find a value of c such that y = x 2 + ecx is a solution of 2y  + y = x.
SOLUTION Let y = x 2 + ecx . Then

y  = 1 + cecx ,

and

2y  + y = 2 1 + cecx + x 2 + ecx = 2 + 2cecx + x 2 + ecx = (2c + 1)ecx + x.

For this to equal x, we must have 2c + 1 = 0, or c = 12 (remember that ecx is never zero).

In Exercises 36, solve using separation of variables.


dy
3. = t 2 y 3
dt
SOLUTION Rewrite the equation as

y 3 d y = t 2 dt.

Upon integrating both sides of this equation, we obtain:


 
y 3 d y = t 2 dt

y4 t3
= + C.
4 3
Thus,
 1/4
4 3
y= t +C ,
3
where C is an arbitrary constant.
4. x yy  = 1 x 2
SOLUTION Rewrite the equation
 
dy 1
xy = 1 x2 as y dy = x d x.
dx x
Upon integrating both sides of this equation, we obtain
   
1
y dy = x dx
x
y2 x2
= ln |x| + C.
2 2
Thus,

y = ln x 2 + A x 2 ,

where A = 2C is an arbitrary constant.


Chapter Review Exercises 1081

dy
5. x y=1
dx
SOLUTION Rewrite the equation as

dy dx
= .
1+ y x
upon integrating both sides of this equation, we obtain
 
dy dx
=
1+y x
ln |1 + y| = ln |x| + C.

Thus,

y = 1 + Ax,

where A = eC is an arbitrary constant.


x y2
6. y  = 2
x +1
SOLUTION Rewrite

dy x y2 dy x
= 2 as = 2 d x.
dx x +1 y2 x +1
Upon integrating both sides of this equation, we obtain
 
dy x
= dx
y2 x2 + 1
1 1  
= ln x 2 + 1 + C.
y 2
Thus,
1
y=1
,
2 ln x + 1 + C
2

where C is an arbitrary constant.

In Exercises 710, solve the initial value problem using separation of variables.

7. y  = cos2 x, y(0) =
4
SOLUTION First, we find the general solution of the differential equation. Because the variables are already separated,
we integrate both sides to obtain
   
1 1 x sin 2x
y = cos2 x d x = + cos 2x d x = + + C.
2 2 2 4

The initial condition y(0) = 4 allows us to determine C = 4 . Thus, the solution is:

x sin 2x
y(x) = + + .
2 4 4


8. y  = cos2 y, y(0) =
4
SOLUTION First, we find the general solution of the differential equation. Rewrite

dy dy
= cos2 y as = d x.
dx cos2 y
Upon integrating both sides of this equation, we find

tan y = x + C;

thus,

y = tan1 (x + C).
1082 C H A P T E R 10 I N T R O D U C T I O N TO D I F F E R E N T I A L E Q U ATI O N S

The initial condition y(0) = 4 allows us to determine the value of C:



= tan1 C so C = 1.
4
Hence, the solution is y = tan1 (x + 1).
9. y  = x y 2 , y(1) = 2
SOLUTION First, we find the general solution of the differential equation. Rewrite

dy dy
= x y2 as = x d x.
dx y2
Upon integrating both sides of this equation, we find
 
dy
= x dx
y2
1 1
= x 2 + C.
y 2
Thus,
1
y=1 .
2x + C
2

The initial condition y(1) = 2 allows us to determine the value of C:


1 2
2=1 =
1 + 2C
2 1 +C
2

1 + 2C = 1
C = 1

Hence, the solution to the initial value problem is


1 2
y=1 = 2 .
x 2
2x 1
2

10. x yy  = 1, y(3) = 2
SOLUTION First, we find the general solution of the differential equation. Rewrite

dy dx
xy =1 as y dy = .
dx x
Next we integrate both sides of the equation to obtain
 
dx
y dy =
x
1 2
y = ln |x| + C.
2
Thus,

y = 2(ln |x| + C).

To satisfy the initial condition y(3) = 2 we must choose the positive square root; moreover,

2 = 2(ln 3 + C) so C = 2 ln 3.

Hence, the solution to the initial value problem is


  
 x2
y = 2(ln |x| + 2 ln 3) = ln + 4.
9
Chapter Review Exercises 1083

11. Figure 1 shows the slope field for y = sin y + t y. Sketch the graphs of the solutions with the initial conditions
y(0) = 1 , y(0) = 0, and y(0) = 1.

2
1
0 t
1
2

2 1 0 1 2
FIGURE 1

SOLUTION

12. Which of the equations (i)(iii) corresponds to the slope field in Figure 2?
(i) y = 1 y 2
(ii) y = 1 + y 2
(iii) y = y 2
SOLUTION From the figure we see that the the slope is positive even for y > 1, thus, the slope field does not correspond
to the equation y = 1 y 2 . Moreover, the slope at y = 0 is positive, so the slope field also does not correspond to the
equation y = y 2 . The slope field must therefore correspond to (ii): y = 1 + y 2 .
13. Let y(t) be the solution to the differential equation with slope field as shown in Figure 2, satisfying y(0) = 0. Sketch
the graph of y(t). Then use your answer to Exercise 12 to solve for y(t).

2
1
0 t
1
2

2 1 0 1 2
FIGURE 2

SOLUTION As explained in the previous exercise, the slope field in Figure 2 corresponds to the equation y = 1 + y 2 .
The graph of the solution satisfying y(0) = 0 is:
1084 C H A P T E R 10 I N T R O D U C T I O N TO D I F F E R E N T I A L E Q U ATI O N S

To solve the initial value problem y = 1 + y 2 , y(0) = 0, we first find the general solution of the differential equation.
Separating variables yields:
dy
= dt.
1 + y2
Upon integrating both sides of this equation, we find

tan1 y = t + C or y = tan(t + C).

The initial condition gives C = 0, so the solution is y = tan x.


14. Let y(t) be the solution of 4 y = y 2 + t satisfying y(2) = 1. Carry out Eulers Method with time step h = 0.05 for
n = 6 steps.
 
1 1
SOLUTION Rewrite the differential equation as y = 4 (y 2 + t) to identify F(t, y) = 4 y 2 + t . With t0 = 2, y0 = 1,
and h = 0.05, we calculate

y1 = y0 + h F(t0 , y0 ) = 1.0375
y2 = y1 + h F(t1 , y1 ) = 1.076580
y3 = y2 + h F(t2 , y2 ) = 1.117318
y4 = y3 + h F(t3 , y3 ) = 1.159798
y5 = y4 + h F(t4 , y4 ) = 1.204112
y6 = y5 + h F(t5 , y5 ) = 1.250361

15. Let y(t) be the solution of (x 3 + 1) y = y satisfying y(0) = 1. Compute approximations to y(0.1), y(0.2), and
y(0.3) using Eulers Method with time step h = 0.1.
SOLUTION Rewriting the equation as y = 3
y
we have F(x, y) = 3y . Using Eulers Method with x0 = 0, y0 = 1
x +1 x +1
and h = 0.1, we calculate
1
y(0.1) y1 = y0 + h F(x 0 , y0 ) = 1 + 0.1 3 = 1.1
0 +1
y(0.2) y2 = y1 + h F(x 1 , y1 ) = 1.209890
y(0.3) y3 = y2 + h F(x 2 , y2 ) = 1.329919

In Exercises 1619, solve using the method of integrating factors.


dy
16. = y + t 2, y(0) = 4
dt
SOLUTION First, we find the general solution of the differential equation. Rewrite the equation as

y y = t 2,

which is in standard form with A(t) = 1 and B(t) = t 2 . The integrating factor is

(t) = e 1 dt = et .
Chapter Review Exercises 1085

When multiplied by the integrating factor, the rewritten differential equation becomes

(et y) = t 2 et .

Integration on both sides (integration by parts is needed for the integral on the right-hand side of the equation) now yields

et y = et (t 2 + 2t + 2) + C;

hence,

y(t) = Cet t 2 2t 2.

The initial condition y(0) = 4 allows us to determine the value of C:

4 = 2 + C so C = 6.

The solution to the initial value problem is then

y = 6et t 2 2t 2.

dy y
17. = + x, y(1) = 3
dx x
SOLUTION First, we find the general solution of the differential equation. Rewrite the equation as

1
y y = x,
x

which is in standard form with A(x) = 1x and B(x) = x. The integrating factor is
 1 1
(x) = e x d x = e ln x = .
x
When multiplied by the integrating factor, the rewritten differential equation becomes
 
1
y = 1.
x
Integration on both sides now yields
1
y = x + C;
x
hence,

y(x) = x 2 + C x.

The initial condition y(1) = 3 allows us to determine the value of C:

3 = 1+C so C = 2.

The solution to the initial value problem is then

y = x 2 + 2x.

dy
18. = y 3t, y(1) = 2
dt
SOLUTION First, we find the general solution of the differential equation. Rewrite the equation as

y  y = 3t,

which is in standard form with A(t) = 1 and B(t) = 3t. The integrating factor is
 
(t) = e A(t) dt = e dt = et .

When multiplied by the integrating factor, the rewritten differential equation becomes

(et y) = 3tet .

Integration on both sides (integration by parts is needed for the integral on the right-hand side of the equation) now yields

et y = (3t + 3)et + C;
1086 C H A P T E R 10 I N T R O D U C T I O N TO D I F F E R E N T I A L E Q U ATI O N S

hence,

y(t) = 3t + 3 + Cet .

The initial condition y(1) = 2 allows us to determine the value of C;

2 = Ce1 + 3(1) + 3 so C = 2e.

The solution to the initial value problem is then

y = 2e et + 3t + 3 = 2et+1 + 3t + 3.

19. y  + 2y = 1 + ex , y(0) = 4
SOLUTION The equation is already in standard form with A(x) = 2 and B(x) = 1 + ex . The integrating factor is

(x) = e 2 d x = e2x .

When multiplied by the integrating factor, the original differential equation becomes

(e2x y) = e2x + e x .

Integration on both sides now yields


1 2x
e2x y = e + e x + C;
2
hence,
1
y(x) = + ex + Ce2x .
2
The initial condition y(0) = 4 allows us to determine the value of C:
1 11
4 = +1+C so C= .
2 2
The solution to the initial value problem is then
1 11
y(x) = + ex e2x .
2 2

In Exercises 2027, solve using the appropriate method.

20. x 2 y  = x 2 + 1, y(1) = 10
SOLUTION First, we find the general solution of the differential equation. Rewrite the equation as

1
y = 1 + 2 .
x
Because the variables have already been separated, we integrate both sides to obtain
  
1 1
y= 1 + 2 d x = x + C.
x x
The initial condition y(1) = 10 allows us to determine the value of C:

10 = 1 1 + C so C = 10.

The solution to the initial value problem is then


1
y=x + 10.
x

21. y  + (tan x)y = cos2 x, y( ) = 2


SOLUTION First, we find the general solution of the differential equation. As this is a first order linear equation with
A(x) = tan x and B(x) = cos2 x, we compute the integrating factor
  1
(x) = e A(x) d x = e tan x d x = e ln cos x = .
cos x
Chapter Review Exercises 1087

When multiplied by the integrating factor, the original differential equation becomes
 
1
y = cos x.
cos x
Integration on both sides now yields
1
y = sin x + C;
cos x
hence,
1
y(x) = sin x cos x + C cos x = sin 2x + C cos x.
2
The initial condition y( ) = 2 allows us to determine the value of C:

2 = 0 + C(1) so C = 2.

The solution to the initial value problem is then


1
y= sin 2x 2 cos x.
2

22. x y  = 2y + x 1, y( 32 ) = 9
SOLUTION First, we find the general solution of the differential equation. This is a linear equation which can be
rewritten as
2 1
y y =1 .
x x

Thus, A(x) = 2x , B(x) = 1 1x and the integrating factor is


  2 1
(x) = e A(x) d x = e x d x = e2 ln x = 2 .
x
When multiplied by the integrating factor, the rewritten differential equation becomes
 
1 1 1
2
y = 2 3.
x x x
Integration on both sides now yields
1 1 1
2
y = + 2 + C;
x x 2x
hence,
1
y(x) = x + + C x 2.
2
 
The initial condition y 32 = 9 allows us to determine the value of C:

3 1 9 40
9= + + C so C= .
2 2 4 9
The solution to the initial value problem is then
40 2 1
y= x x+ .
9 2

23. (y 1)y  = t, y(1) = 3


SOLUTION First, we find the general solution of the differential equation. This is a separable equation that we rewrite
as

(y 1) d y = t dt.

Upon integrating both sides of this equation, we find


 
(y 1) d y = t dt
1088 C H A P T E R 10 I N T R O D U C T I O N TO D I F F E R E N T I A L E Q U ATI O N S

y2 1
y = t2 + C
2 2
y 2 2y + 1 = t 2 + C
(y 1)2 = t 2 + C

y(t) = t 2 + C + 1

To satisfy the initial condition y(1) = 3 we must choose the negative square root; moreover,

3 = 1 + C + 1 so C = 15.

The solution to the initial value problem is then



y(t) = t 2 + 15 + 1

y + 1 y  = ytet ,
2
24. y(0) = 1.
SOLUTION First, we find the general solution of the differential equation. This is a separable equation that we rewrite
as
 
1 1 2
+ d y = tet dt.
y y

Upon integrating both sides of this equation, we find


   
1 1 2
+ d y = tet dt
y y
1 2
2 y + ln y = et + C.
2
Note that we cannot solve explicitly for y(t). The initial condition y(0) = 1 still allows us to determine the value of C:
1 3
2(1) + ln 1 = +C so C= .
2 2
Hence, the general solution is given implicitly by the equation
1 2 3
2 y + ln y = e x + .
2 2

dw 1 + w2
25. =k , w(1) = 1
dx x
SOLUTION First, we find the general solution of the differential equation. This is a separable equation that we rewrite
as
dw k
= d x.
1 + w2 x
Upon integrating both sides of this equation, we find
 
dw k
= dx
1+w 2 x
tan1 w = k ln x + C
w(x) = tan(k ln x + C).

Because the initial condition is specified at x = 1, we are interested in the solution for x > 0; we can therefore omit the
absolute value within the natural logarithm function. The initial condition w(1) = 1 allows us to determine the value of
C:

1 = tan(k ln 1 + C) so C = tan1 1 = .
4
The solution to the initial value problem is then
 
w = tan k ln x + .
4

3y 1
26. y  + =t +2
t
Chapter Review Exercises 1089

SOLUTION We rewrite this first order linear equation in standard form:

3 1
y + y =t +2+ .
t t

Thus, A(t) = 3t , B(t) = t + 2 + 1t , and the integrating factor is



(t) = e A(t) dt = e3 ln t = t 3 .

When multiplied by the integrating factor, the rewritten differential equation becomes

(t 3 y) = t 4 + 2t 3 + t 2 .

Integration on both sides now yields


1 5 1 4 1 3
t3y = t + t + t + C;
5 2 3
hence,
1 2 1 1 C
y(t) = t + t + + 3.
5 2 3 t

y
27. y  + = sin x
x
SOLUTION This is a first order linear equation with A(x) = 1x and B(x) = sin x. The integrating factor is

(x) = e A(x) d x = eln x = x.

When multiplied by the integrating factor, the original differential equation becomes

(x y) = x sin x.

Integration on both sides (integration by parts is needed for the integral on the right-hand side) now yields

x y = x cos x + sin x + C;

hence,
sin x C
y(x) = cos x + + .
x x

28. State whether the differential equation can be solved using separation of variables, the method of integrating factors,
both, or neither.
(a) y  = y + x 2 (b) x y  = y + 1
(c) y  = y 2 + x 2 (d) x y  = y 2
SOLUTION

(a) The equation y  = y + x 2 is a first order linear equation; hence, it can be solved by the method of integration factors.
However, it cannot be written in the form y  = f (x)g(y); therefore, separation of variables cannot be used.
(b) The equation x y  = y + 1 is a first order linear equation; hence, it can be solved using the method of integration
factors. We can rewrite this equation as y  = 1x (y + 1); therefore, it can also be solved by separating the variables.
(c) The equation y  = y 2 + x 2 cannot be written in the form y  = f (x)g(y); hence, separation of variables cannot be
used. This equation is also not linear; hence, the method of integrating factors cannot be used.
(d) The equation x y  = y 2 can be rewritten as y  = 1x y 2 ; therefore, it can be solved by separating the variables. Since
it is not a linear equation, the method of integrating factors cannot be used.
dy
29. Let A and B be constants. Prove that if A > 0, then all solution of + Ay = B approach the same limit as t .
dt
SOLUTION This is a linear first-order equation in standard form with integrating factor

(t) = e A dt = e At .

When multiplied by the integrating factor, the original differential equation becomes

(e At y) = Be At .
1090 C H A P T E R 10 I N T R O D U C T I O N TO D I F F E R E N T I A L E Q U ATI O N S

Integration on both sides now yields


B At
e At y = e + C;
A
hence,
B
y(t) = + CeAt .
A
Because A > 0,
 
B B
lim y(t) = lim + CeAt = .
t t A A

We conclude that if A > 0, all solutions approach the limit BA as t .


30. The trough in Figure 3 is filled with water. At time t = 0 (in seconds), water begins leaking through a circular hole
at the bottom of radius 3 in. Let y(t) be the water height at time t. Find a differential equation for y(t) and solve it to
determine when the trough will be half empty.

8 ft
4 ft

6 ft
12 ft

FIGURE 3

SOLUTION y(t) obeys the differential equation:

dy Bv(y)
= ,
dt A(y)
where v(y) denotes the velocity of the water flowing through the hole when the trough is filled to height y, B denotes the
area of the hole and A(y) denotes the area of the horizontal cross section of the trough at height y. By Torricellis Law,
 2
v(y) = 8 y ft/s. The area of the hole is B = 12 3 = 16 ft2 . The horizontal cross section of the trough at height

y is a rectangle of length 12 and width w(y). As w(y) varies linearly from 6 when y = 0 to 8 when y = 4, it follows that
y
w(y) = + 6;
2
therefore, the area of horizontal cross section at height y is
y 
A(y) = 12w(y) = 12 + 6 = 6y + 72 = 6(y + 12).
2
The differential equation for y(t) then becomes

dy y
= .
dt 12(y + 12)
Rewrite the above equation as
y + 12
d y = dt,
y 12

and then integrate both sides to obtain


 
y + 12
dy = dt
y 12
2 3/2
y + 24y 1/2 = t + C.
3 12
Thus,
t
y 3/2 + 36y 1/2 = + C.
8
The initial condition y(0) = 4 allows us to determine the value of C:

43/2 + 36 41/2 = 0 + C so C = 80.


Chapter Review Exercises 1091

Finally, the height of the water is given implicitly by the equation


t
y 3/2 + 36y 1/2 = + 80.
8

The volume of the trough is (8+6)4


2 12 = 336. Hence, the trough is half empty when

(w(y) + 6)y
12 = 168
2
y 
+ 12 y = 28
2
y 2 + 24y 56 = 0

The roots of this quadratic
equation are y = 12 10 2. We eliminate the negative root, and thus the trough is half
empty when y = 10 2 12. The height of the water takes on this value when
 3/2  1/2
10 2 12 + 36 10 2 12 = t + 80.
8
Solving for t, we find t 61.56 seconds.
31. Find the solution of the logistic equation y = 0.4y(4 y) satisfying y(0) = 8.
SOLUTION We can write the given equation as
 y
y = 1.6y 1 .
4
This is a logistic equation with k = 1.6 and A = 4. Therefore,
A 4
y(t) = = .
1 ekt /C 1 e1.6t /C
The initial condition y(0) = 8 allows us to determine the value of C:
4 1 1
8= ; 1 = ; so C = 2.
1 C1 C 2

Thus,
4 8
y(t) = = .
1 e1.6t /2 2 e1.6t

32. Let y(t) be the solution of y = 0.3y(2 y) with y(0) = 1. Determine lim y(t) without solving for y explicitly.
t
SOLUTION We write the given equation in the form
 y
y = 0.6y 1 .
2
This is a logistic equation with A = 2 and k = 0.6. Because the initial condition y(0) = y0 = 1 satisfies 0 < y0 < A,
the solution is increasing and approaches A as t . That is, lim y(t) = 2.
t
33. Suppose that y  = ky(1 y/8) has a solution satisfying y(0) = 12 and y(10) = 24. Find k.
SOLUTION The given differential equation is a logistic equation with A = 8. Thus,

8
y(t) = .
1 ekt /C
The initial condition y(0) = 12 allows us to determine the value of C:
8 1 2
12 = ; 1 = ; so C = 3.
1 C1 C 3

Hence,
8 24
y(t) = kt
= .
1 e /3 3 ekt
Now, the condition y(10) = 24 allows us to determine the value of k:
24
24 =
3 e10k
1092 C H A P T E R 10 I N T R O D U C T I O N TO D I F F E R E N T I A L E Q U ATI O N S

3 e10k = 1
ln 2
k= 0.0693.
10

34. A lake has a carrying capacity of 1,000 fish. Assume that the fish population grows logistically with growth constant
k = 0.2 days1 . How many days will it take for the population to reach 900 fish if the initial population is 20 fish?
SOLUTION Let y(t) represent the fish population. Because the population grows logistically with k = 0.2 and A =
1000,
1000
y(t) = .
1 e0.2t /C
The initial condition y(0) = 20 allows us to determine the value of C:
1000 1 1
20 = ; 1 = 50; so C = .
1 C1 C 49

Hence,
1000
y(t) = .
1 + 49e0.2t
The population will reach 900 fish when
1000
= 900.
1 + 49e0.2t
Solving for t, we find

t = 5 ln 441 30.44 days.

35. A rabbit population on a deserted island increases exponentially with growth rate k = 0.12 months1 . When
the population reaches 150 rabbits (say, at time t = 0), hunters begin killing the rabbits at a rate of r rabbits per month.
(a) Find a differential equation satisfied by the rabbit population P(t).
(b) How large can r be without the rabbit population becoming extinct?
SOLUTION
(a) The rabbit population P(t) obeys the differential equation
dP
= 0.12P r,
dt
where the term 0.12P accounts for the exponential growth of the population and the term r accounts for the rate of
decline in the rabbit population due to hunting.
(b) Rewrite the linear differential equation from part (a) as
dP
0.12P = r,
dt
which is in standard form with A = 0.12 and B = r . The integrating factor is
 
(t) = e A dt = e 0.12 dt = e0.12t .

When multiplied by the integrating factor, the rewritten differential equation becomes

(e0.12t P) = r e0.12t .

Integration on both sides now yields


r 0.12t
e0.12t P = e + C;
0.12
hence,
r
P(t) = + Ce0.12t .
0.12
The initial condition P(0) = 150 allows us to determine the value of C:
r r
150 = +C so C = 150 .
0.12 0.12
Chapter Review Exercises 1093

The solution to the initial value problem is then


 r  0.12t r
P(t) = 150 e + .
0.12 0.12
r < 0, then lim P(t) = , and the population becomes extinct. Therefore, in order for the
Now, if 150 0.12
t
population to survive, we must have
r
150 0 or r 18.
0.12
We conclude that the maximum rate at which the hunters can kill the rabbits without driving the rabbits to extinction is
r = 18 rabbits per month.

1  1 y2
36. Show that y = sin(tan x + C) is the general solution of y = . Then use the addition formula for the
1 + x2
(cos C)x + sin C
sine function to show that the general solution may be written y =  .
1 + x2
SOLUTION Rewrite

dy 1 y2 dy dx
= as  = .
dx 1 + x2 1 y2 1 + x2

Upon integrating both sides of this equation, we find


 
dy dx
 =
1y 2 1 + x2

sin1 y = tan1 x + C

Thus,

y(x) = sin tan1 x + C .

To express the solution in the required form, we use the addition formula

sin( + ) = sin cos + sin cos

This yields

y(x) = sin tan1 x cos C + sin C cos tan1 x .

Using the figure below, we see that



x
sin tan1 x =  ; and
1 + x2

1
cos tan1 x =  .
1 + x2
Finally,
x cos C sin C (cos C)x + sin C
y=  + =  .
1 + x2 1 + x2 1 + x2

1 + x2 x

tan1 x

37. A tank contains 100 gal of pure water. Water is pumped out at a rate of 20 gal/min, and polluted water with
a toxin concentration of 0.1 lb/gal is pumped in at a rate of 15 gal/min. Let y(t) be the amount of toxin present in the
tank at time t.
1094 C H A P T E R 10 I N T R O D U C T I O N TO D I F F E R E N T I A L E Q U ATI O N S

(a) Find a differential equation satisfied by y(t).


(b) Solve for y(t).
(c) Plot y(t) and find the time t at which the amount of toxin is maximal.
SOLUTION
(a) Because water flows into the tank at a rate of 15 gallons per minute but flows out at a rate of 20 gallons per minute,
there is a new outflow of 5 gallons per minute from the tank. Therefore, at any time t, there are 100 5t gallons of water
in the tank. Now, to determine the differential equation satisfied by y(t), we employ the basic rate balance
dy
= toxin rate in toxin rate out.
dt
The amount of toxin per minute coming into the tank is
lb
toxin rate in = concentration water rate in = 0.1 15 = 1.5 .
min
To determine the toxin rate out we first compute the toxin concentration at time t:
lbs of toxin y(t)
toxin concentration = = .
gallons of water 100 5t
Since water flows out at a rate of 20 gallons per minute,
y 4y
toxin rate out = 20 = .
100 5t 20 t
Thus, the differential equation is
dy 4y
= 1.5 .
dt 20 t
(b) Rewrite the linear first-order differential equation found in part (a) as
dy 4 3
+ y= .
dt 20 t 2
The integrating factor for this equation is
 1 1
(t) = e4 (20t) dt = .
(20 t)4
When multiplied by the integrating factor, the rewritten differential equation becomes
 
1 3
y = .
(20 t)4 2(20 t)4
Integration on both sides now yields
1 1
y= + C;
(20 t)4 2(20 t)3
hence,
20 t
y(t) = + C(20 t)4 .
2
The initial condition y(0) = 0 allows us to determine the value of C:
1
0 = 10 + 204 C so C = .
16000
Therefore,
20 t (20 t)4
y(t) = .
2 16000
(c) A plot of y(t) is shown below.

5
4
3
2
1

5 10 15 20
Chapter Review Exercises 1095

To determine the time at which the amount of toxin is maximal, we note that

dy d 20 t (20 t)4 1 (20 t)3
= = + =0
dt dt 2 16000 2 4000

when

3
t = 20 10 2 7.4 minutes.

You might also like